NUR 110 Exam 5

Pataasin ang iyong marka sa homework at exams ngayon gamit ang Quizwiz!

Servant Leadership

A type of leadership based on humble service to all God's people.

Which word is best described by the following: the protection and support of another's rights?

Advocacy

Harriet accepted money from a client as a "thank you" gift. Her employer found out and fired her.

American Nurses Association Code of Ethics

For prevention of Hepatitis A, you decided to conduct health education activities. Which of the following is Irrelevant? A. Use of sterile syringes and needles B. Safe food preparation and food handling by vendors C. Proper disposal of human excreta and personal hygiene D. Immediate reporting of water pipe leaks and illegal water connections

Answer: (A) Hepatitis A is transmitted through the fecal oral route. Hepatitis B is transmitted through infected body secretions like blood and semen.

To determine the possible sources of sexually transmitted infections, which is the BEST method that may be undertaken by the public health nurse? A. Contact tracing B. Community survey C. Mass screening tests D. Interview suspects

Answer: (A) Contact tracing is the most practical and reliable method of finding possible sources of person-to-person transmitted infections, such as sexually transmitted diseases.

"Public health services are given free of charge". Is this statement true or false? A. The statement is true; it is the responsibility of government to provide haste services B. The statement is false; people pay indirectly for public health services C. The statement may be true or false; depending on the Specific service required D. The statement may be true or false; depending on policies of the government concerned.

Answer: (B) Community health services, including public health services, are prepaid paid services, through taxation, for example.

Which of the following is an epidemiologic function of the nurse during an epidemic? A Conducting assessment of suspected cases to detect the communicable diseases B. Monitoring the condition of the cases affected by the communicable disease C. Participating in the investigation to determine the source of epidemic D. Teaching the community on preventive measures against the disease

Answer: (C) Epidemiology is the study of patterns of occurrence and distribution of disease in the community, as well as the factors that affect disease patterns. The purpose of an epidemiologic investigation is to identify the source of an epidemic, i.e., what brought about the epidemic.

1. Which of the following is true of long-term care facilities? A. They provide care for homeless adults. B. They provide care to people of any age. C. They provide care only for people with dementia. D. They provide care only to older adults.

B. They provide care to people of any age.

One of the most effective ways nurses may use statistics is to: A. determine daily staffing to maintain the best patient care. B. predict the hospital census for the following year. C. evaluate the outcomes of prenatal care provided. D. compute the number of women who conceive each year.

C. evaluate the outcomes of prenatal care provided. Rationale: Because statistics examine trends in large groups of people, they can help evaluate the quality of care, including prenatal care. (12)

Nurses practicing in a critical care unit must acquire specialized skills and knowledge to provide care to the critically ill client. These nurses can validate this specialty competence through what process?

Certification

A nurse provides client care within a philosophy of ethical decision making and professional expectations. What is the nurse using as a framework for practice?

Code of Ethics A professional code of ethics provides a framework for making ethical decisions and sets forth professional expectations. Codes of ethics inform both nurses and society of the primary goals and values of the profession.

The nurse is assigned to care for the following patients. Based on information received at the change-of-shift report, which patient should the nurse assess first? A. A 1-hour-old neonate with a heart rate of 130 bpm B. A postpartum mother requiring discharge information C. A breastfeeding mother with reports of painful breasts D. A patient recovering from cesarean section reporting red, warm right calf.

D. A patient recovering from cesarean section reporting red, warm right calf. Rationale: A red, warm calf may indicate phlebitis, a possibly life-threatening condition that must be immediately addressed. (12)

Your class project is to prepare a speech on the functions of humor in nursing situations for your peers. Following the presentation, you know your message was understood by the participants when they respond that humor: Increases the social distance between persons and assists in putting people at ease Facilitates learning Holds in anger and aggression Aggravates coping mechanisms

Facilitates learning

Herbal Supplements

Fewer and less severe side effects than prescription medications Take control over own health Preventative treatment St. John's wort, peppermint, chamomile, aloe vera, cranberry, echinacea, ginger, etc. ** Can have side effects and interact with prescription medications**

An HIV-positive client discovers that his name is published in a research report on HIV care prepared by his nurse. He is hurt and files a lawsuit against her. Which offense has the nurse committed?

Invasion of privacy

What is the legal source of rules of conduct for nurses

Nurse Practice Acts

Good Samaritan Law

Provides limited protection to someone who voluntarily chooses to provide first aid

The newly graduated nurse is supervising licensed practical nurses and certified nursing assistants. It is important to understand the role of each person.

Scope of Practice

Calvin is considering a transfer to the interventional radiology department. He's unfamiliar with the expectations in the nursing role within that department and looks for practice guidelines.

Standard of Practice

Before meeting with a client with a terminal illness, a new graduate nurse reviews information on spirituality. Which is the best explanation of spirituality? That which gives people purpose and meaning in their lives. A formalized religious dogma. A nondenominational community service. People being responsible for their life patterns.

That which gives people purpose and meaning in their lives.

A registered nurse has had her license suspended after being convicted of being impaired at work. What governing body has the authority to revoke or suspend a nurse's license

The State Board of Nurse Examiners

A nervous client scheduled for heart surgery the next day calls his pastor. During this conversation, he asks the pastor to pray for him. After completing the encounter, the client calmly says, "Now I know I'll be in good hands." This sense of calmness by the client can be explained through which of the following? The action has increased his well-being. The action has decreased his well-being. The action has a questionable action on his well-being. The action assured him people will know he is sick.

The action has a questionable action on his well-being.

What is included in the ANA Code of Ethics for Nurses?

The nurse advances the profession by actions such as meeting continuing education requirements. • The nurse advocates for clients and encourages clients to advocate for themselves. • The nurse delivers care in a nonjudgmental manner that is sensitive to client diversity. • The nurse takes action to promote safe and appropriate care environments.

What factors contribute to a person's ethical beliefs? Select all that apply. Career Values Attitudes Beliefs Morals

Values Attitudes Beliefs Morals

The parish nurse may perform which of these actions? a. Assist in finding a doctor or filing health claims b. Provide immunizations at a low-income clinic c. Care for someone who is dying d. Complete disability assessments

a. Assist in finding a doctor or filing health claims

Therapeutic Touch

an alternative therapy that involves using one's hands to consciously direct an energy exchange from the practitioner to the patient to facilitate healing or pain relief

The global health nurse will often see which of these conditions? Select all that apply. a. Metabolic syndrome b. Dehydration c. Parasitic infections d. Insect-related illness e. Malnutrition

b. Dehydration c. Parasitic infections d. Insect-related illness e. Malnutrition

Teaching a class on smoking cessation is what type of nursing intervention? a. Primary b. Secondary c. Tertiary d. Occupational

b. secondary As smoking is already the problem, teaching a smoking cessation class is considered a second level, or secondary interventions

13. Which nursing action demonstrates the category of utility in the informatics competency classification system? a. computer and software use b. maintenance of privacy c. critical thinking and evidence-based practice d. application of accountability and quality assurance

c. critical thinking and evidence based practice

Which action most clearly demonstrates a nurse's commitment to social justice?

lobbying for an expansion of Medicare eligibility and benefits

Scientific Management

the application of scientific principles to increase efficiency in the workplace

Your friend is considering in-vitro fertilization in hopes of becoming pregnant. Which one of the following is an accurate statement? "There is some evidence that acupuncture improves the chance of pregnancy in this situation." "Massage therapy may increase your sense of relaxation, which may help in getting pregnant." "Ask your doctor about which herbs will increase the likelihood of pregnancy." "Research suggests that yoga improves the chance of pregnancy in this situation."

"There is some evidence that acupuncture improves the chance of pregnancy in this situation."

A client asks how herbs are similar to prescribed medications. What is the nurse's best answer? "They are nothing alike. You should ask your doctor these types of questions." "Thirty percent of current prescription drugs are derived from plants." "Medications are much more effective than herbs." "Herbs are more dangerous than prescribed medications."

"Thirty percent of current prescription drugs are derived from plants."

Nurses practice the professional value of autonomy when providing nursing care for clients. Which nursing actions best describe the use of this value?

-A nurse reads The Patient Care Partnership to a visually impaired client. • A nurse collaborates with other health care team members to ensure the best possible treatment for a client.

Biomedical Perspective: Allopathy

-Conventional Western medicine/treatment -Treats pathologies and symptoms

1. The nurse hears a client calling out for help, hurries down the hallway to the client's room, and finds the client lying on the floor. The nurse performs an assessment, assists the client back to bed, notifies the health care provider of the incident, and completes an incident report. Which statement should the nurse document on the incident report? a. The client fell out of bed b. The client climbed over the side rails c. The client was found lying on the floor d. The client became restless and tried to get out of bed.

1. C- The incident report should contain the client's name, age, and diagnosis. The report should contain a factual description of the incident, any injuries experienced by those involved, and the outcome of the situation. The correct option is the only one that describes the facts as observed by the nurse. Options 1, 2, and 4 are interpretations of the situation and are not factual information as observed by the nurse.

10. Nursing staff members are sitting in the lounge taking their morning break. An unlicensed assistive personnel (UAP) tells the group that she thinks that the unit secretary has acquired immunodeficiency syndrome (AIDS) and proceeds to tell the nursing staff that the secretary probably contracted the disease from her husband, who is supposedly a drug addict. Which legal tort has the UAP violated? a. Libel b. Slander c. Assault d. Negligence

10. B- Defamation is a false communication or a careless disregard for the truth that causes damage to someone's reputation, either in writing (Libel) or verbally (slander). An assault occurs when a person puts another person in fear of a harmful or an offensive contact. Negligence involves the actions of professionals that fall below standard of care for a specific professional group

11. An 87-year-old woman is brought to the emergency department for treatment of a fractured arm. On assessment, the nurse notes old and new ecchymotic areas on the client's chest and legs and asks the client how the bruises were sustained. The client, although reluctant, tells the nurse in confidence that her son frequently hits her if supper is not prepared on time when he arrives home from work. What is the most appropriate nursing response? a. "Oh really I will discuss this situation with your son" b. "Let's talk about the ways you can manage your time to prevent this from happening" c. "Do you have any friends that can help you out until you resolve these important issues with your son?" d. "As a nurse, I am legally bound to report abuse. I will stay with you while you give the report and help find a safe place for you to stay.

11. D- The nurse must report situations related to child or elder abuse, gunshot wounds and other criminal acts, and certain infectious diseases. Confidential issues are not to be discussed with nonmedical personnel or the client's family or friends without the client's permission. Clients should be assured under a legal obligation. Option 1, 2, and 3 do not address the legal implications of the situation and do not ensure a safe environment for the client.

12. The nurse calls the health care provider (HCP) regarding a new medication prescription because the dosage prescribed is higher than the recommended dosage. The nurse is unable to locate the HCP, and the medication is due to administered. Which action should the nurse take? a. Contact the nursing supervisor b. Administer the dose prescribed c. Hold the medication until the HCP can be contacted d. Administer the recommended dose until the HCP can be located

12. A- If the HCP writes a prescription that requires clarification, the nurse's responsibility is to contact the HCP. If there is no resolution regarding the prescription because the HCP cannot be located or because the prescription remains as it was written after talking with the HCP, the nurse should contact the nurse manager or nursing supervisor for further clarification as to what the next step should be. Under no circumstances should the nurse proceed to carry out the prescription until obtaining clarification.

13. The nurse employed in a hospital is waiting to receive a report from the laboratory via the facsimile (fax) machine. The fax machine activates and the nurse expects the report, but instead receives a sexually oriented photograph. Which is the most appropriate nursing action? a. Call the police b. Cut up the photograph and throw it away c. Call the nursing supervisor and report the incident d. Call the laboratory and ask for the individual's name who sent the photograph

13. C- Ensuring a safe workplace is a responsibility of an employing institution. Sexual harassment in the workplace is prohibited by state and federal laws. Sexually suggestive jokes, touching, pressuring a co-worker for a date, and open displays of or transmitting sexually oriented photographs or posters are examples of conducts that could be considered sexual harassment by another worker. If the nurse believes that he or she is being subjected to unwelcome sexual conduct, these concerns should be reported to the nursing supervisor immediately. Option 1 is unnecessary at this time. Options 2 and 4 are inappropriate initial actions.

2. A client is brought to the emergency department by emergency medical services (EMS) after being hit by a car. The name of the client is unknown, and the client has sustained a severe head injury and multiple fractures and is unconscious. An emergency craniotomy is required. Regarding informed consent for the surgical procedure, which is the best action? a. Obtain a court order for the surgical procedure b. Ask the EMS team to sign the informed consent c. Transport the victim to the operating room for surgery d. Call the police to identify the client and locate the family.

2. C- In general, there are two situations in which informed consent of an adult client is not needed. One is when an emergency is present and delaying treatment for the purpose of obtaining informed consent would result in injury or death to the client. The second is when the client waives the right to give informed consent. Option 1 will delay emergency treatment, and option 2 is inappropriate. Although option 4 may be pursued, it is not the best action

3. The nurse has just assisted a client back to bed after a fall. The nurse and health care provider have assessed the client and have determined that the client is not injured. After completing the incident report, the nurse should implement which action net? a. Reassess the client b. Conduct a staff meeting to describe the fall c. Document in the nurse's notes that an incident report was completed. d. Contact the nursing supervisor to update information regarding the fall

3. A- After a client's fall, the nurse must frequently reassess the client because potential complications do not always appear immediately after the fall. The client's fall should be treated as private information and shared on a "need to know" basis. Communication regarding the event should involve only the individuals participating in the client's care. An incident report is a problem-solving document; however, its completion is not documented in the nurse's notes. If the nursing supervisor has been made aware of the incident, the supervisor will contact the nurse if status update is necessary.

4. The nurse arrives at work and is told to report (float) to the intensive care unit (ICU) for the day because the ICU is understaffed and needs additional nurses to care for the clients. The nurse has never worked in the ICU. The nurse should take which action first? a. Call the hospital lawyer b. Refuse to float to the ICU c. Call the nursing supervisor d. Identify tasks that can be performed safely in the ICU

4. D- Floating is an acceptable legal practice used by hospitals to solve understaffing problems. Legally, the nurse cannot refuse to float unless a union contract guarantees that nurses can work only in a specified area or the nurse can prove the lack of knowledge for the performance of assigned tasks. When encountering this situation, the nurse should set priorities and identify potential areas of harm to the client. The nursing supervisor is called if the nurse is expected to perform tasks that he or she cannot safely perform. Calling the hospital lawyer is a premature action.

5. The nurse who works on the night shift enters the medication room and finds a co-worker with a tourniquet wrapped around the upper arm. The co-worker is about to insert a needle, attached to a syringe containing clear liquid, in the antecubital area. Which is the most appropriate action by the nurse? a. Call security b. Call the police c. Call the nursing supervisor d. Lock the co-worker in the medication room until help is obtain

5. C- Nurse practice acts require reporting impaired nurses. The board of nursing has jurisdiction over the practice of nursing and may develop plans for treatment and supervision of the impaired nurse. This incident needs to be reported to the nursing supervisor, who will then report to the board of nursing and other authorities, such as the police, as required. The nurse may call security if a disturbance occurs, but no information in the question supports this need, and so this is not the appropriate action. Option 4 is an inappropriate and unsafe action.

6. A hospitalized client tells the nurse that a living will is being prepared and that the lawyer will be bringing the will to the hospital today for witness signatures. The client asks the nurse for assistance obtaining a witness to the will. Which is the most appropriate response to the client? a. "I will sign as a witness to your signature." b. "You will need to find a witness on your own.' c. "Whoever is available at the time will sign as a witness for you." d. "I will call the nursing supervisor to seek assistance regarding your request."

6. D- Living wills, also known as natural death acts in some states, are required to be in writing and signed by the client. The client's signature must be witnessed by specified individuals or notarized. Laws and guidelines regarding living wills vary from state to state, and it is the responsibility of the nurse to know the laws. Many states prohibit any employee, including the nurse of a facility where the client is receiving care, from being a witness. Option 2 is nontherapeutic and not a helpful response. The nurse should seek the assistance of the nursing supervisor.

7. The nurse has made an error in a narrative documentation of an assessment finding on a client and obtains the client's record to correct the error. The nurse should take which action to correct the error? a. Documenting a late entry into the client's record b. Trying to erase the error for space to write in the correct data c. Using whiteout to delete the error to write in the correct data d. Drawing one line through the error, initialing and dating, and then documenting the correct information.

7. D- If the nurse makes an error in narrative documentation in the client's record, the nurse should follow agency policies to correct the error. This includes drawing one line through the error, initializing and dating the line, and then documenting the correct information. A late entry is used to document additional information not remembered at the initial time of documentation. Erasing data from the client's record and the use of whiteout are prohibited

8. Which identifies accurate nursing documentation notations? Select all that apply a. The client slept through the night b. Abdominal wound dressing is dry and intact without drainage c. The client seemed angry when awakened for vital sign measurement d. The client appears to become anxious when it is time for respiratory treatments e. The client's left lower medial leg wound is 3 cm in length without redness, drainage, or edema

8. A, B, E- Factual documentation contains descriptive, objective information about what the nurse sees, hears, feels, or smells. The use of inferences without supporting factual data is not acceptable because it can be misunderstood. The use of vague terms, such as seemed or appears is not acceptable because these words suggest that the nurse is stating an opinion.

9. A nursing instructor delivers a lecture to nursing students regarding the issue of client's rights and asks a nursing student to identify a situation that represents an example of invasion of client privacy. Which situation, if identified by the student, indicates an understanding of a violation of this client right? a. Performing a procedure without consent b. Threatening to give a client a medication c. Telling the client that he or she cannot leave the hospital d. Observing care provided to the client without the client's permission

9. D- Invasion of privacy occurs with unreasonable intrusion into an individual's private affairs. Performing a procedure without consent is an example of battery. Threatening to a give a client a medication constitutes assault. Telling the client that the client cannot leave the hospital constitutes false imprisonment.

A community health nurse is concerned about poor compliance with recommended immunization schedules for infants (birth to 2 years). After surveying some of the parents, it is determined there are many variables inhibiting compliance, such as limited clinic hours, difficulty making appointments through an automatic system, and no reminder call/email or follow-up intervention. When meeting with staff, managers, and physicians to resolve the issues, it is revealed there are many underlying issues that must be addressed, such as an insufficient number of providers available to treat this demographic. This nurse is acting in what capacity? A. Collaborator B. Counselor C. Case manager D. Client Advocate

A. Collaborator the nurse is acting with the team of providers and health-care workers to come up with a solution for problem, which is collaboration

While working in a local clinic 3 days per week, a nurse recognizes that there is a rise in teen pregnancy. On days off, the nurse works with teachers and schools to develop programs that will help keep pregnant teens in school and also increase awareness of teen pregnancy. What type of nursing is this? A. Community-oriented B. Client advocate C. Counseling D. Educator

A. Community-oriented recognizes needs in the community and work collaboratively with those in position to make changes

Which individuals played a role in decreasing infections for patients, improving the safety and the health of mother and child during and after birth? (Select all that apply) A. Crede B. Saint Vincent de Paul C. Lister D. Pasteur E. Semmelweis

A. Crede C. Lister D. Pasteur E. Semmelweis Rationale: These four individuals all played a role in decreasing infection. (2)

What are considered components of a community? Select all that apply. A. Structure B. Status C. Process D. Culture E. Resources F. Vulnerability

A. Structure general characteristics such as demographics and data on health-care services make up the structure of community B. Status biological, emotional, and social outcome components make up the status of a community C. Process overall effectiveness level of the community and includes ability to interact, solve problems, and the value one places on his or her position within the community

What is true of family-centered care? (Select all that apply) A. The nurse's role is to enter into a partnership with the family. B. The health care professionals are the primary decision makers. C. Families are considered vulnerable and require protection in regard to patient status. D. Families contribute their ability to accept and maintain control over the health care of family members.

A. The nurse's role is to enter into a partnership with the family. D. Families contribute their ability to accept and maintain control over the health care of family members. Rationale: Family-centered care recognizes the strength of the family as the core of planning and implementing health care. The family as caregivers and decision makers is integral to this care. The family accepts and maintains control over the health care of family members, and the nurse's role is to enter into a contract or partnership with the family to achieve the goals of health for its members. The other answer options contradict the definition of family-centered care. (6)

Which healing modalities would be least appropriate for the patient receiving chemotherapy to use? A.Acupuncture B.Meditation C.Biofeedback D.Yoga

A.Acupuncture

Lisa is an RN who transferred to the pediatrics unit 2 months ago. She noted the low morale and conflict among the nurses based on the manager's authoritarian leadership style. What are Lisa's best actions? Select all that apply. A.Talk with the manager in private about the unit's needs. B.Discuss various leadership styles with the nurses. C.Ask the nursing supervisor to talk with the manager about her leadership style. D.Do nothing and continue to observe for improvements. E.Organize a staff meeting so the nurses can voice their concerns to the manager.

A.Talk with the manager in private about the unit's needs. B.Discuss various leadership styles with the nurses

7. Computerized provider order entry (CPOE) allows orders to be directly communicated to the appropriate department. Other advantages of CPOE include: a. decrease in number of transcribing errors. b. enhanced provider acceptance because of new technology. c. decreased work flow issues in general. d. less dependence on technology and computers.

ANS: A Computerized provider order entry (CPOE) allows orders to be directly communicated to the appropriate department. CPOE systems ensure legible orders and have the potential to reduce ordering and transcribing errors. Disadvantages of CPOE include workflow issues, provider resistance to new technology, and overdependence on technology (AHRQ, 2012).

9. Computerized provider order entry (CPOE): a. allows orders to be communicated to the appropriate department. b. creates an intermediary for order transcription. c. slows documentation and provider communication. d. may lead to increased ordering and transcription errors.

ANS: A Computerized provider order entry (CPOE) allows orders to be directly communicated to the appropriate department—diet orders to dietary, medication orders to the pharmacy, laboratory orders to the laboratory. Elimination of an intermediary for order transcription decreases the potential for errors related to the ambiguity of handwritten orders and allows quicker responses by appropriate departments. Legibility and availability of computerized documentation improve provider communication. The Agency for Healthcare Research and Quality (AHRQ) recommends CPOE as one of the safe practices for better health care. CPOE systems ensure legible orders and have the potential to reduce ordering and transcribing errors. Disadvantages of CPOE include workflow issues, provider resistance to new technology, and overdependence on technology (AHRQ, 2012).

1. The integration of nursing, computers, and information science for the management and communication of data, information, knowledge, and wisdom is: a. nursing informatics. b. computer science. c. medical informatics. d. informatics.

ANS: A Informatics is a broad academic field encompassing artificial intelligence, cognitive science, computer science, information science, and social science. Medical informatics refers to informatics related to health care and describes a distinct specialty in the discipline of medicine. Nursing informatics is a specialty area of informatics that addresses the use of health information systems to support nursing practice. The American Nurses Association (ANA, 2008) states that the specialty of nursing informatics integrates nursing computer and information science for the management and communication of data, information, knowledge, and wisdom.

15. One classification system for nursing informatics competencies uses technical, utility, and leadership categories. Leadership competencies involve: a. maintaining privacy and confidentiality. b. using computers and other technological equipment. c. using a variety of software programs. d. addressing critical thinking applications.

ANS: A One classification system for nursing informatics competencies uses technical, utility, and leadership categories. Leadership competencies address the ethical and management issues related to using IT in nursing practice, education, research, and administration. Specific leadership competencies include the application of accountability, maintenance of privacy and confidentiality, and quality assurance. Technical competencies pertain to the use of computers and other technological equipment and the use of a variety of software programs for word processing, spreadsheet and database development, presentation, referencing, and e-mail. Utility competencies address critical thinking and evidence-based practice applications. Nurses who have a utility competency recognize the relevance of nursing data for improving practice and can access multiple information sources for gathering evidence for clinical decision making.

4. The home health nurse provides care for a patient with congestive heart failure. Daily the patient weighs himself and takes his own temperature, pulse, respirations and blood pressure. That information is sent as electronic data to the patient's physician and nurse daily to make adjustments to the plan of care as indicated. This is an example of: a. telehealth nursing. b. computerized decision support system (DSS). c. computerized provider order entry (CPOE). d. point of care technology.

ANS: A Telehealth nursing is the transmission by a nurse of electronic data, images, or audio from a patient's bedside or home to other health providers for the purpose of providing care and improving outcomes. Patients may have telehealth hardware in their homes to provide in-home monitoring and direct reporting to their health care providers. Computerized decision support systems (DSSs) include safe practice alerts and reminders that improve the quality of care. Some DSSs assist in determining a correct diagnosis and choosing an appropriate medication. Computerized provider order entry (CPOE) allows orders to be directly communicated to the appropriate department—diet orders to dietary, medication orders to the pharmacy, laboratory orders to the laboratory. Computers, tablets, or pocket devices used at the bedside for documentation are examples of point-of-care technology. Patient data collected by a nurse and recorded electronically are immediately available to all members of the health care team.

16. The Computer Ethics Institute has developed guidelines for ethics in the development and use of computer technologies. These guidelines are called: a. the Ten Commandments of Computer Ethics. b. the eHealth Code of Ethics. c. HIPAA guidelines. d. the Internet Healthcare Coalition.

ANS: A The Computer Ethics Institute (CEI) was founded in 1985 to serve as a forum and resource for identifying, assessing, and responding to ethical issues associated with the advancement of information technologies and to facilitate the recognition of ethics in the development and use of computer technologies. CEI developed the Ten Commandments of Computer Ethics. The eHealth Code of Ethics, developed by Health Informatics Europe (2005), is "to ensure that people worldwide can confidently and with full understanding of known risks realize the potential of the Internet in managing their own health and the health of those in their care." The Health Insurance Portability and Accountability Act (HIPAA) of 1996 sets the standards on how security and confidentiality of health care information must be maintained. The act also sets the penalties for any breach in security of health care data. The Internet Healthcare Coalition, founded in 1997, was created to promote quality health resources online and to ensure that consumers and professionals are able to find reliable, quality information online.

8. When using electronic medical records (EMR), the nurse knows that the EMR: a. holds the documentation of a single episode of care. b. is a longitudinal record of care for each patient. c. is widely used for individual health care encounters. d. includes progress notes for all disciplines.

ANS: A The electronic medical record (EMR), which is the documentation of a single episode of care (i.e., outpatient visit or inpatient stay), becomes a part of the electronic health record (EHR), which is a longitudinal record of care. EHRs are becoming widely used for individual health care encounters and for maintaining patients' health records over long periods. As EHRs become fully implemented, they include provider order entries, progress notes for all disciplines, computerized medication profiles, access to diagnostic test results on a timely basis, decision support systems, and online clinical reminders and alerts.

18. While adopting new technology to enhance patient care and safety, nurses can continue to provide: a. compassionate care. b. consumer empowerment. c. self-management of wellness. d. education about health care.

ANS: A While adopting new technology that enhances patient care and safety, it is the nurse's responsibility to continue to provide compassionate care. The spread of IT supports consumer empowerment and self-management of wellness and disease. Consumers can electronically access their own health information, communicate with their health care providers, and seek needed education about health care.

1. Nurses at the beginner level of informatics competencies stress the need to (Select all that apply.) a. be computer literate. b. be Internet literate. c. become a software developer. d. be able to suggest areas for IT system improvement.

ANS: A, B Beginner skills include computer, information, and web literacy; fundamental skills in information management and computer technology; and the ability to identify and collect relevant data. The nurse at the beginning level may have keyboarding skills, can document in the EHR, and can look up medications and other health information on reputable Internet reference sites. Developing new software to enhance nursing care is at the specialist level of competency. Being able to suggest areas for IT system improvement is at the experienced level of competency.

3. It is unethical to (Select all that apply.) a. use proprietary software for which you have not paid. b. assume a false identity. c. access your own patient's records. d. take another person's intellectual output and claim it as your own.

ANS: A, B, D It is not ethical to use proprietary software for which you have not paid. It is not ethical to assume a false identity. It is not ethical to take another person's intellectual output and claim it as your own. It is ethical to access your own patient's files for legitimate work purposes.

2. Which are major areas to be addressed when evaluating a website? (Select all that apply.) a. Authority b. Purpose c. Popularity d. Verification

ANS: A, B, D Website evaluation criteria include authority, purpose, coverage, currency, objectivity, accuracy, and verification. Popularity is not one of the criteria to use when evaluating a website.

1. The use of telemonitoring offers the opportunity to: (Select all that apply.) a. reduce cost of health care. b. improve patient satisfaction. c. increase duplicate orders. d. improve patient outcomes. e. improve organization.

ANS: A, B, D, E The use of telemonitoring offers the opportunity to reduce the cost of health care while improving outcomes and patient satisfaction. Use of health care IT has improved organization, communication, and decision making; reduced duplicate orders, charting time, and paperwork; made medication administration safer; and enhanced information access and administrative functions.

3. In reviewing a patient's written chart, the nurse notes the use of the terms "bed sore," "decubitus ulcer." And "pressure ulcer." The nurse knows in order to reach maximum potential in computerized charting and data analysis that a standardized nursing terminology must be utilized. In an attempt to standardize nursing terminology: (Select all that apply.) a. The Nursing Minimum Data Set (NMDS) was the first attempt to do so b. The focus was to provide a shared understanding of patient problem labels. c. The NMDS data was completed and is the definitive source of patient labels. d. The ICNP was developed to provide a standard for international nurses. e. Standardized terminology can lead to better utilization of resources.

ANS: A, B, E A standardized nursing terminology is a structured vocabulary that provides a common means of communication among nurses. A standardized language ensures that when a nurse talks about a specific patient problem, another nurse fully understands the problem. An example is the choice between pressure ulcer, decubitus ulcer, and bedsore. Do all nurses in all settings have a shared understanding of these labels for a patient problem? The Nursing Minimum Data Set (NMDS) represents the first attempt to standardize the collection of essential nursing data. These core data, used on a regular basis by most nurses in the delivery of care across settings, provide accurate descriptions of the nursing diagnoses, nursing care, outcomes of care, and nursing resources used. Collected on an ongoing basis, the NMDS enables nurses to compare data across populations, settings, geographic areas, and time. The International Classification for Nursing Practice (ICNP), developed under the auspices of the International Council of Nurses (ICN), provides a standard that facilitates the description and comparison of nursing practice locally, regionally, nationally, and internationally.

4. The Health Insurance Portability and Accountability Act (HIPAA) of 1996: (Select all that apply.) a. requires the user to have verification codes. b. ensures access to information without fear of audits. c. sets the standards on how information is maintained. d. sets the penalties for any breach in security of health data. e. has no legal authority relative to security issues.

ANS: A, C, D Access to electronic records requires a user to have system access and verification codes as a measure of security and protection of the patient's privacy. The codes leave an electronic trail of authorized users that can be audited. HIPAA sets the standards on how security and confidentiality of health care information must be maintained. The act also sets the penalties for any breach in security of health care data.

2. The Technology Informatics Guiding Education Reform (TIGER) initiative identified a set of skills needed by all nurses practicing in the 21st century. The TIGER Vision Pillars include: (Select all that apply.) a. management and leadership. b. certification by HIMSS. c. communication and Collaboration. d. informatics design. e. IT policy and culture.

ANS: A, C, D, E The Technology Informatics Guiding Education Reform (TIGER) initiative (2012) identified a set of skills needed by all nurses practicing in the 21st century. The TIGER Vision Pillars include management and leadership, education, communication and collaboration, informatics design, and IT policy and culture. This skill set includes informatics competencies that range from basic computer skills to advanced-level IT and literacy competencies and expertise. TIGER's vision for the future of nursing addresses informatics and emerging technologies to provide safer, patient-centered care by using evidence and technology in practice, education, and research. After meeting the educational and experience requirements, the nurse can receive certification in nursing informatics from the Health Care Information and Management Systems Society (HIMSS) and through the American Nurses Credentialing Center (ANCC). However, certification is not one of TIGER's Vision Pillars.

3. Standardized nursing terminologies include a. the ABC coding system. b. NANDA, NIC, and NOC. c. SNOMED-CT. d. ICD9 codes.

ANS: B A standardized nursing terminology is a structured vocabulary that provides a common means of communication among nurses. A standardized language ensures that when a nurse talks about a specific patient problem, another nurse fully understands the problem. Standardized nursing terminologies recognized by the American Nurses Association include NMDS (nursing minimum data set), NMMDS (nursing minimum data set), CCC (clinical care classification), ICNP (international classification of nursing practice), NANDA-I (NANDA International), NIC (nursing interventions classification), NOC (nursing outcomes classification), Omaha System, and PNDS (perioperative nursing data set). ABC and SNOMED CT are examples of multidisciplinary terminologies. ICD-9 details terminology for billing and coding, not nursing.

1. Data are best described as a. organized information. b. facts, observations, and measurements. c. knowledge used to make decisions. d. the appropriate application of knowledge.

ANS: B Data are facts, observations, and measurements that can be used as a basis for reasoning, discussion, or calculation.

5. Information can be best defined as a. organized and processed information that can be applied to problem solving and decision making. b. organized and processed data that can be communicated and are meaningful and useful to the recipient. c. the transformation of patient data to clinical information. d. the appropriate application of knowledge.

ANS: B Information is organized and processed data that can be communicated and are meaningful and useful to the recipient. Knowledge is organized and processed information that can be applied to problem solving and decision making. The transformation of patient data to clinical information becomes nursing knowledge. Wisdom addresses the use of knowledge and experience to manage and solve problems. Wisdom is the appropriate application of knowledge.

8. The standard of how security and confidentiality of health care information must be maintained is determined by a. NANDA-I. b. HIPAA. c. each health care facility. d. the Computer Ethics Institute.

ANS: B The Health Insurance Portability and Accountability Act (HIPAA) of 1996 sets the standards on how security and confidentiality of health care information must be maintained. Health care facilities may have additional guidelines to follow, but they do not set the standard. NANDA International has developed nursing terminology related to nursing diagnoses. The Computer Ethics Institute has developed its own Ten Commandments of Computer Ethics, but they do not set the standard.

11. The nurse can see data relationships, can make judgments based on trends and patterns in the data, is skilled in information management and the use of computer technology, and is able to suggest areas for IT system improvement. The nurse's level of informatics competency can be described as: a. beginner. b. experienced. c. specialist. d. innovator.

ANS: B Descriptions of nursing informatics competencies often focus on levels that include beginner, experienced, specialist, and innovator. Beginner skills include computer, information, and web literacy; fundamental skills in information management and computer technology; and the ability to identify and collect relevant data. The nurse at the beginning level may have keyboarding skills, can document in the EHR, and look up medications and other health information on reputable Internet reference sites. The nurse at the experienced level of informatics competencies is able to see data relationships and make judgments based on trends and patterns in data, is skilled in information management and the use of computer technology, and is able to suggest areas for IT system improvement. The nurse at the specialist level of competency focuses on information needs for the practice of nursing; integrates and applies information science, computer science, and nursing science; and applies skills in critical thinking, data management, processing, and system development. At the specialist level of competency, the nurse may conduct research based on information trends or patient data, devise applications for computer technology in nursing, or develop new software to enhance nursing care. Nursing informatics innovators conduct research and generate theory. They develop solutions and understand the interdependence of systems, disciplines, and outcomes.

2. The hospital has recently implemented computer charting. The computerization of nursing practice: a. enhances and increases the time spent on documentation. b. makes patient data immediately available to the health care team. c. makes retrieval of data more difficult but safer. d. is enhanced by limiting the use of point-of-care technology.

ANS: B Patient data collected by a nurse and recorded electronically are immediately available to all members of the health care team. The computerization of nursing practice data enables capture, storage, retrieval, organization, processing, and analysis of information. The information can be used to make a diagnosis, plan for care, provide nursing decision support, enhance documentation, and identify nursing care trends and costs. Systems that support data collection at the point of care can directly enhance patient care by decreasing the time spent on documentation, reducing the potential for errors, and supporting improved assessment and data communication. Computers, tablets, or pocket devices used at the bedside for documentation are examples of point-of-care technology.

4. Social media includes such sites as (Select all that apply.) a. HIPAA. b. Twitter. c. Facebook. d. LinkedIn.

ANS: B, C, D Social media includes online technologies such as Facebook, Twitter, and LinkedIn, which allow people to communicate easily via the Internet to share information and resources. HIPAA stands for the Health Insurance Portability and Accountability Act, and is not an example of social media.

4. Information Literacy is best defined as a. knowing how to access an electronic library. b. understanding data processing. c. knowing how to recognize when information is needed and to locate, evaluate, and effectively use the needed information. d. knowledge of computers and the ability to use them efficiently.

ANS: C Information literacy is the ability to recognize when information is needed and to locate, evaluate, and effectively use the needed information. Computer literacy is knowledge of computers and the ability to use them efficiently. Understanding data processing and knowing how to access an electronic library are not the best definitions of information literacy.

12. The director of nursing on a medical-surgical floor has met education and experience requirements in nursing informatics. The nurse might expect administration to request that he/she pursue: a. technical competencies. b. utility competencies. c. certification from ANCC. d. leadership competencies.

ANS: C After meeting the educational and experience requirements, the nurse can receive certification in nursing informatics from the Health Care Information and Management Systems Society (HIMSS) and through the American Nurses Credentialing Center (ANCC). Technical competencies pertain to the use of computers and other technological equipment and the use of a variety of software programs for word processing, spreadsheet and database development, presentation, referencing, and e-mail. Utility competencies address critical thinking and evidence-based practice applications. Nurses who have a utility competency recognize the relevance of nursing data for improving practice and can access multiple information sources for gathering evidence for clinical decision making. Leadership competencies address the ethical and management issues related to using IT in nursing practice, education, research, and administration. Specific leadership competencies include the application of accountability, maintenance of privacy and confidentiality, and quality assurance. Technical, utility, and leadership competencies can be achieved without certification.

5. Information technology (IT) can be used to increase patient safety. The nurse uses IT in this way by: a. creating redundancy in orders making them safer. b. removing the need for verification by the nurse. c. analyzing errors to develop prevention strategies. d. eliminating the need for bar codes in medication administration.

ANS: C IT can be used to increase patient safety. Errors are analyzed to develop strategies for prevention. Diagnostic test results are available faster to support treatment decisions and avoid redundancy in orders. When technology such as a bar-code medication administration (BCMA) system is used as part of the process of medication administration, fewer errors are made. After signing into the system or scanning his/her identification (ID) badge, the nurse electronically scans the bar codes of the patient ID, the medication administration record (MAR), and the drug to determine that the right patient is getting the right drug and dose at the right time. An alert signals a potential error, and it is the nurse's responsibility to verify all information before administration

10. The nurse is providing care to a patient newly diagnosed with multiple sclerosis. The patient expresses the desire to communicate with other people living with the disorder. The nurse appropriately refers the patient to: a. an e-mail list with the patient's contacts. b. a social media blog. c. a listserv concerning multiple sclerosis. d. Facebook, Twitter, and LinkedIn.

ANS: C Listservs can be used in health care to connect groups of patients with common problems or to send updated information to large groups. E-mail has become a common means of communication but would not be focused on the patient's issues. Social media include online technologies such as Facebook, Twitter, and LinkedIn that allow people to communicate easily by the Internet to share information and resources, but they are more general than listservs. These technologies enable a potentially massive community of participants to collaborate, providing a mechanism for tapping into collective power in ways previously unachievable. A blog is a social medium that is usually maintained by an individual and has regular entries of commentary, descriptions of events, or other material such as graphics or videos. Most blogs are interactive, allowing visitors to leave comments and message each other. Many blogs focus on health care issues.

14. The patient asks the nurse about how to evaluate websites and standards used to evaluate Internet health sites. The nurse appropriately refers the patient to: a. World Health Organization. b. the U.S. Food and Drug Administration. c. the Internet Healthcare Coalition. d. the U.S. Federal Trade Commission.

ANS: C The Internet Healthcare Coalition, founded in 1997, was created to promote quality health resources online and to ensure that consumers and professionals are able to find reliable, quality information online. This organization consults with various government agencies, including the World Health Organization, the U.S. Food and Drug Administration, and the U.S. Federal Trade Commission, on efforts to promote credible online health care information and combat online health fraud.

6. When technology such as a bar-code medication administration (BCMA) system is used as part of the process of medication administration, fewer errors are made. The proper procedure when using the BCMA includes: a. signing into the system using the patient's ID number. b. typing in the patient's name and room number. c. scanning the patient's ID, MAR, and medication. d. discontinuing the medication if the system signals an error.

ANS: C When technology such as a BCMA system is used as part of the process of medication administration, fewer errors are made. After signing into the system or scanning his/her ID badge, the nurse electronically scans the bar codes of the patient ID, the MAR, and the drug to determine that the right patient is getting the right drug and dose at the right time. An alert signals a potential error, and it is the nurse's responsibility to verify all information before administration.

2. A listserv is best described as a. a website. b. an online library source. c. a blog. d. a program that sends messages to multiple e-mail addresses.

ANS: D A listserv is a computer program that automatically sends messages to multiple e-mail addresses on a mailing list. Listservs can be used in health care to connect groups of patients with common problems or to send updated information to large groups.

6. Computer Literacy is best defined as a. knowing how to access an electronic library. b. understanding data processing. c. knowing how to recognize when information is needed and to locate, evaluate, and effectively use the needed information. d. knowledge of computers and the ability to use them efficiently.

ANS: D Computer literacy is knowledge of computers and the ability to use them efficiently. Information literacy is the ability to recognize when information is needed and to locate, evaluate, and effectively use the needed information. Understanding data processing and knowing how to access an electronic library are not the best definitions of computer literacy.

7. A global perspective on nursing practice might best be obtained from a. NANDA, NIC, and NOC. b. ICD9 Codes. c. SNOMED-CT. d. ICNP.

ANS: D The International Classification for Nursing Practice (ICNP), developed under the auspices of the International Council of Nurses (ICN), provides a standard that facilitates the description and comparison of nursing practice locally, regionally, nationally, and internationally. NANDA, NIC, and NOC are examples of standardized nursing terminologies. SNOMED CT is an example of multidisciplinary terminology. ICD9 is used for billing purposes.

13. Patients frequently seek sources for health information online, and nurses, as advocates, need to be prepared to help patients evaluate online sources. To do this, the nurse asks who sponsors the site, is the author listed, and the author's credentials. The nurse is evaluating what? a. Purpose b. Coverage c. Currency d. Authority

ANS: D Authority involves knowing who the sponsor or publisher is. Is this a personal page? Where does it come from? Is the author or organization listed? What are the author's credentials? Purpose is determining to focus of the site. Does the site inform? Explain? Share? Disclose? Sell? What is the intended audience? Coverage tries to determine if citations are correct. Is there a balance of text and images? Currency refers to when the site was created. How often is it updated?

3. Nurses working surrounded by computers and mobile IT must develop skills in the use of all available technology. At the same time, it is important to recognize that: a. the technology in use today will be the same tomorrow. b. cell phones are not usually allowed in the acute care setting. c. most forms of mobile technology are in violation of HIPAA guidelines. d. the technology supports bedside and remote charting.

ANS: D Nurses working surrounded by computers and mobile IT must develop skills in the use of all available technology. At the same time, it is important to recognize that the rapid advancement of IT means that the technology in use today may be entirely different tomorrow. Some facilities have computer access at every bedside, and others have mobile computers, sometimes called workstations on wheels (WOWs), that can be taken to each bedside. Nurses using technology as part of patient care need to work within facility policy and HIPAA guidelines. The technology supports bedside and remote charting. Nurses may use a portable device such as a smartphone or tablet computer to access reference materials, including medical information and vast amounts of drug information. Some facilities issue these devices to staff.

17. The focus of nursing informatics is: a. direct patient care. b. increasing documentation time. c. the introduction of different EHRs. d. how patient care can be improved.

ANS: D The focus of nursing informatics is not on direct patient care but on how the process of patient care can be improved and patient safety ensured. There are challenges to advancements in IT, such as increased amount of time spent documenting, cost of equipment, protection of patient privacy, use of different EHRs in different facilities, and resistance by health care providers to new technology.

A client asks the nurse the differences between traditional therapies and alternative therapies. What is the best response? Alternative therapies cost less than traditional therapies. Alternative therapies are used if traditional therapies are ineffective. Alternative therapies can be as effective as traditional therapies for some conditions. Alternative therapies utilize products from nature but traditional therapies do not.

Alternative therapies can be as effective as traditional therapies for some conditions.

The nurse, Suzie, has an allergy to latex. Her employer replaced all latex products used on the unit with non-latex substitutions so Suzie will not get sick.

Americans With Disabilities Act (ADA)

Which is the BEST control measure for AIDS? A. Being faithful to a single sexual partner B. Using a condom during each sexual contact C. Avoiding sexual contact with commercial sex workers D. Making sure that one's sexual partner does not have signs of AIDS

Answer: (A) Sexual fidelity rules out the possibility of getting the disease by sexual contact with another infected person. Transmission occurs mostly through sexual intercourse and exposure to blood or tissues.

As an epidemiologist, the nurse is responsible for reporting cases or notifiable diseases. What law mandates reporting cases of notifiable diseases? A. Act 3573 B. RA.3753 C. RA 1054 D. RA 1082

Answer: (A) Act 3573, the Law on Reporting of Communicable Diseases, enacted in 1929, mandated the reporting of diseases listed in the law to the nearest health station.

Among children aged 2 months to 3 years, the most prevalent form of meningitis is caused by which microorganism? A. Hemophilus Influenzae B. Morbillivirus C. Streptococcus Pneumoniae D. Neisseria meningitides

Answer: (A) Hemophilus meningitis is unusual over the age of 5 years. In developing countries, the peak incidence is in children less than 6 months of age. Morbillivirus is the etiology of measles. Streptococcus pneumoniae and Neisseria meningitidis may cause meningitis, but age distribution is not specific in young children.

What is the best course of action when there is a measles epidemic in a nearby municipality? A. Give measles vaccine to babies aged 6 to 3 months B. Give babies aged 6 to 11 months one dose of 100,000 IU of Retinol C. Instruct mother to keep their babies at home to prevent disease transmission D. Instruct mothers to feed their babies adequately to enhance their babies resistance

Answer: (A) Ordinarily, measles vaccine is given at 9 months of age. During an impending epidemic, however, one dose may be given to babies aged 6 to 8 months. The mother is instructed that the baby needs another dose when the baby is 9 months old.

According to C.E. Winslow, which of the following is the goal of Public Health? A. For people to attain their birthrights of health and longevity B. For promotion of health and prevention and diseases C. For people to have access to basic health services D. For people to be organized in their health efforts

Answer: (A) According to Winslow, all public health efforts are for people to realize their birthrights of health and longevity.

An indicator of success in community organizing is when people are able to: A. Participate in community activities for the solution of a community problem B. Implement activities for the solution of the community problem C. Plan activities for the solution of the community problem D. Identify the health problem as a common concern

Answer: (A) Participation in community activities in resolving a community problem may be in any of the processes mentioned in the other choices.

In traditional Chinese medicine, the yielding, negative and feminine force is termed: A. Yin B. Yang C. Qi D. Chai

Answer: (A) Yang is the male dominating, positive and masculine force.

Which is the primary goal of community health nursing? A. To support and supplement the efforts of the medical profession in the promotion of health and prevention of B. To enhance the capacity of individuals, families and communities to cope with their health needs C. To increase the productivity of the people by providing them with services that will increase their level of health D. To contribute to national development through promotion of family welfare, focusing particularly on mothers and children

Answer: (B)

Which of the following is the mission of the Department of Health? A. Health for all Filipinos B. Ensure the accessibility and quality of health C. Improve the general health status of the population D. Health in the hands of the Filipino people by the year 2020 D. Appropriateness

Answer: (B)

The following are strategies implemented by the DOH to prevent mosquito-borne diseases. Which of these is most effective in the control of Dengue fever? A. Stream seeding with larva-eating fish B. Destroying breeding places of mosquitoes C. Chemoprophylaxis of non-immune persons going to endemic areas D. Teaching people in endemic areas to use chemically treated mosquito nets

Answer: (B) Aedes aegypti, the vector of Dengue fever, breeds in stagnant, clear water. Its feeding time is usually during the daytime. It has a cyclical pattern of occurrence, unlike malaria which is endemic in certain parts of the country.

A mother brought her 10 month old infant for consultation because of fever which started 4 days prior to consultation. To determine malaria risk, what will you do? A. Do a tourniquet test B. Ask where the family resides C. Get a specimen for blood smear D. Ask if the fever is present everyday

Answer: (B) Because malaria is endemic, the first question to determine malaria risk is where the client's family resides. If the area of residence is not a known endemic area, ask if the child had traveled within the past 6 months, where he/she was brought and whether he/she stayed overnight in that area.

Which of the following is an advantage of a home visit? A. It allows the nurse to provide nursing care to a greater number of people B. It provides an opportunity to do first hand appraisal of the home situation C. It allows sharing of experience among people with similar health problems D. It develops the family's initiative in providing for health needs of its members

Answer: (B) Choice A is not correct since a home visit requires that the nurse spend so much time with the family. Choice C is an advantage of a group conference, while choice D is true of a clinic consultation.

CHN (Community Health Nursing) is a community-based practice. Which best explains this statement? A. The service is provided in the natural environment of people B. The nurse has to conduct community diagnosis to determine nursing needs and problems C. The service are based on the available resources within the community D. Priority setting is based on the magnitude of the health problems identified

Answer: (B) Community-based practice means providing care to people in their own natural environments: the home, school and workplace, for example.

The most frequent causes of death among clients with AIDS are opportunistic diseases. Which of the following opportunistic infections is characterized by tonsilllopharyngitis? A. Respiratory candidiasis B. Infectious mononucleosis C. Cytomegalovirus disease D. Pneumocystis carinii pneumonia

Answer: (B) Cytomegalovirus disease is an acute viral disease characterized by fever, sore throat and lymphadenopathy.

To improve compliance to treatment, what innovation is being implemented in DOTS (Directly Observed Treatment Short-Course)? A. Having the health worker follow up the client at home B. Having the health worker or a responsible family member monitor drug intake C. Having the patient come to the health center every month to get his medications D. Having a target list to check on whether the patient has collected his monthly supply of drugs

Answer: (B) Directly Observed Treatment Short Course is so-called because a treatment partner, preferably a health worker accessible to the client, monitors the client's compliance to the treatment.

According to Freeman and Heinrich, community health nursing is a developmental service. Which of the following best illustrates this statement? A. The community health nurse continuously develops himself personally and professionally B. Health education and community organizing are necessary in providing community health services C. Community health nursing in intended primarily for health promotion and prevention and treatment of disease. D. The goal of community health nursing is to provide nursing services to people in their own place of .residence

Answer: (B) The community health nurse develops the health capability of people through health education and community organizing activities.

Which immunization produces a permanent scar? A. DPT (Diphtheria, Pertussis, Tetanus) B. BCG (Bacillus Calmette-Guérin) C. Measles vaccination D. Hepatitis B vaccination

Answer: (B) BCG (Bacillus Calmette-Guérin) causes the formation of a superficial abscess, which begins 2 weeks after immunization. The abscess heals without treatment, with the formation of a permanent scar. BCG Immunity or protection against tuberculosis (TB). The vaccine may be given to persons at high risk of developing TB

A business firm must employ an occupational health nurse when it has at least how many employees. A. 21 B. 101 C. 201 D. 301

Answer: (B) Based on R.A. 1054, an occupational nurse must be employed when there are 30 to 100 employees and the workplace is more than 1 km. away from the nearest health center.

RA 1054 is also known as the Occupational Health Act. Aside from the number of employees, what other factor must be considered in determining the occupational health privileges to which the workers will be entitled? A. Type of occupation,: agriculture, commercial, industrial B. Location of the workplace in relation to health facilities C. Classification of the business enterprise based on net profit D. Sex and age composition of employees

Answer: (B) Based on R.A. 1054, an occupational nurse must be employed when there are 30 to 100 employees and the workplace is more than 1 km. away from the nearest health center.

Which of the following signs indicates the need for sputum examination for AFB (Acid Fast Bacillus)? A. Hematemesis B. Fever for 1 week C. Cough for 3 weeks D. Chest pain for 1 week

Answer: (C) A client is considered a PTB (Pulmonary Tuberculosis) suspect when he has cough for 2 weeks or more, plus one or more of the following signs: fever for 1 month or more; chest pain lasting for 2 weeks or more not attributed to other conditions; progressive, unexplained weight loss; night sweats; and hemoptysis

One of the participants in a hilot training class asked you to whom she should refer a patient in labor who develops a complication. You will answer, to the; A. Public health nurse B. Rural health midwife C. Municipal health officer (Physician) D. Any of these health professionals

Answer: (C) A public health nurse and rural health midwife can provide care during normal childbirth. A physician should attend to a woman with a complication during labor.

Mosquito-borne diseases are prevented mostly with the use of mosquito control measures. Which of the following is NOT appropriate for malaria control? A. Use of chemically treated mosquito nets B. Seeding of breeding places with larva-eating fish C. Destruction of breeding places of the mosquito vector D. Use of mosquito-repelling soaps, such as those with basil or citronella

Answer: (C) Anopheles mosquitoes breed in slow-moving, clear water, such as mountain streams.

Population- focused nursing practice requires which of the following processes? A. Community organizing . B. Nursing, process C. Community diagnosis D. Epidemiologic process

Answer: (C) Population-focused nursing care means providing care based on the greater need of the majority of the population. The greater need is identified through community diagnosis.

In the year 1980, the World Health Organization declared the Philippines, together with some other countries in the Western Pacific Region, "free" of which disease? A. Pneumonic plaque B. Poliomyelitis C. Small pox D. Anthrax

Answer: (C) The last documented case of Smallpox was in 1977 at Somalia.

Which type of family-nurse contact will provide you with the best opportunity to observe family dynamics? A. Clinic consultation B. Group conferences C. Home visit D. Written communication

Answer: (C) Dynamics of family relationships can best be observed in the family's natural environment, which is the home.

Which is CONTRARY to the principles in planning a home visit? A. A home visit should have a purpose of objective B. The plan should revolve around the family health .needs C. A home visit should be conducted in the manner prescribed by RHU D. Planning of continuing care should involve a responsible-family member

Answer: (C) The home visit plan should be flexible and practical, depending on factors, such as the family's needs and the resources available to the nurse and the family.

Secondary prevention for malaria includes? A. Planting of neem or eucalyptus trees B. Residual spraying of insecticides at night C. Determining whether a place is endemic or not D. Growing larva-eating fish in mosquito breeding places

Answer: (C) This is diagnostic and therefore secondary level prevention. The other choices are for primary prevention.

A 32 year old client came for consultation at the health center with the chief complaint of fever for a week. Accompanying symptoms were muscle pains and body malaise. A week after the start of fever, the client noted yellowish discoloration of his sclera. History showed that he waded in flood waters about 2 weeks before the onset of symptoms. Based on this history/ which disease condition will you suspect? A. Hepatitis A B. Hepatitis B C. Tetanus D. Leptospirosis

Answer: (D) A bacterial disease spread through the urine of infected animals. Leptospirosis is transmitted through contact with the skin or mucous membrane with water or moist soil contaminated with urine of infected animals, like rats.

You are the PHN in the city health center. A client underwent screening for AIDS using ELISA. His result was positive. What is the best course of action that you may take? A. Get a thorough history of the client, focusing on the practice of high risk behavior B. Ask the client to be accompanied by a significant person before revealing the result. C. Refer the client to the physician since he is the best person to reveal the result to the client D. Refer the client for a supplementary test, such as Western blot, since the ELISA result maybe false

Answer: (D) A client having a reactive ELISA result must undergo a more specific test, such as Western blot. A negative supplementary test result means that the ELISA result was false and that, most probably, the client is not infected.

Which clients are considered targets for DOTS (Directly Observed Treatment Short-course) category? A. Sputum negative cavitary cases B. Clients returning after default C. Relapses and failures of previous PTB treatment regimens D. Clients diagnosed for the first time through a positive sputum exam

Answer: (D) Category I is for new clients diagnosed by sputum examination and clients diagnosed to have a serious form of extrapulmonary tuberculosis, such as TB osteomyelitis.

A barangay had an outbreak of German measles. To prevent congenital rubella, what is the BEST advice that you can give to women in the first trimester of pregnancy in the barangay? a. Advice them on the sign of German Measles b. Avoid crowded places, such as markets and moviehouses c. Consult at the health center where rubella vaccine may be given d. Consult a physician who may give them rubella immunoglobulin

Answer: (D) Rubella vaccine is made up of attenuated German measles viruses. This is contraindicated in pregnancy. Immune globulin, a specific prophylactic against German measles, may be given to pregnant women.

Antiretroviral agents, such as AZT are used in the management of AIDS. Which of the following is not an action expected of these drugs? A. They prolong the life of the client with AIDS B. They reduce the risk of opportunistic infections C. They shorten the period of communicability of the disease D. They are able to bring about a cure of the disease condition

Answer: (D) There is no known treatment for AIDS. Antiretroviral agents reduce the risk of opportunistic infections and prolong life, but does not cure the underlying immunodeficiency.

The public health nurse takes an active role in community participation. What is the primary goal of community organizing? A. To educate the people regarding community health problems B. To mobilize the people to resolve community health problems C. To maximize the community's resources in dealing with health problems

Answer: (D) Community organizing is a developmental service, with the goal of developing the people's self-reliance in dealing with community health problems. A, B and C are objectives of contributory objectives to this goal.

Which of the following is a function of epidemiology? A. Identifying the disease condition based on manifestations presented by a client B. Determining factors that contributed to the occurrence of pneumonia in a 3 year old C. Determining the efficacy of the antibiotic used in the treatment of the 3 year old client with pneumonia D. Evaluating the effectiveness of the implementation of the Integrated Management of Childhood Illness

Answer: (D) Epidemiology is used in the assessment of a community or evaluation of interventions in community health practice.

Primary health care is a total approach to community development. Which of the following is an indicator of success in the use of the primary health care approach? A. Health services are provided free of charge to individuals and families B. Local officials are empowered as the major decision makers in matters of health C. Health workers are able too provide care based on identified health needs of the people D. Health programs are sustained according to the level of development of the community

Answer: (D) Primary health care is essential health care that can be sustained in all stages of development of the community.

You are a new B.S.N. graduate. You want to become a Public Health Nurse. Where will you apply? A. Department of Health B. Provincial Health Office C. Regional Health Office D. Rural Health Unit

Answer: (D) R.A. 7160 devolved basic health services to local government units (LGU's). The public health nurse is an employee of the LGU.

Sputum examination is the major screening tool for pulmonary tuberculosis. Clients would sometimes get false negative results in this exam. This means that the test is not perfect in terms of which characteristic of a diagnostic examination? A. Effectiveness B. Efficacy C. Specificity D. Sensitivity

Answer: (D) Sensitivity is the capacity of a diagnostic examination to detect cases of the disease. If a test is 100% sensitive, all the cases tested will have a positive result, i.e., there will be no false negative results

Which of the following is the most prominent feature of public health nursing? A. It involves providing home care to sick people who are not confined in the hospital B. Services are provided free of charge to people within the catchment area C. The public health nurse functions as part of a team providing a public health nursing service D. Public health nursing focuses on preventive, not curative services

Answer: (D) The catchment area in PHN consists of a residential community, many of whom are well individuals who have greater need for preventive rather than curative services.

Which of the following women should be considered as special targets for family planning? A. Those who have two children or more B. Those with medical conditions such as anemia C. Those younger than 20 years and older than 35 years D. Those who just had a delivery within the past 15 months

Answer: (D) The ideal birth spacing is at least two years. 15 months plus 9 months of pregnancy = 2 years.

Among the following diseases, which is airborne? A. Viral conjunctivitis B. Acute poliomyelitis C. Diphtheria D. Measles

Answer: (D) Viral conjunctivitis is transmitted by direct or indirect contact with discharges from infected eyes. Acute poliomyelitis is spread through the fecal-oral route and contact with throat secretions, whereas diphtheria is through direct and indirect contact with respiratory secretions.

In correcting misconception and myths about certain diseases and their management, the health worker should first: A. Identify the myths and misconceptions prevailing in the community B. Identify the source of these myths and misconceptions C. Explain how and why these myths came about D. Select appropriate strategies to correct them.

Answer: A

An ambulatory care nurse is discussing preoperative procedures with a Japanese-American client who is scheduled for surgery the following week. During the discussion, the client continually smiles and nods the head. The nurse interprets this nonverbal behavior as: A. Reflecting a cultural value B. An acceptance of the treatment C. The client is agreeable to the required procedures D. The client understands the preoperative procedures

Answer: A Nodding or smiling by a Japanese-American client may reflect only the cultural value of interpersonal harmony. This nonverbal behavior may not be an indication of agreement with the speaker, an acceptance of the treatment, or an understanding of the procedure.

Katherine is a young Unit Manager of the Pediatric Ward. Most of her staff nurses are senior to her, very articulate, confident and sometimes aggressive. Katherine feels uncomfortable believing that she is the scapegoat of everything that goes wrong in her department. Which of the following is the best action that she must take? A. Identify the source of the conflict and understand the points of friction B. Disregard what she feels and continue to work independently C. Seek help from the Director of Nursing D. Quit her job and look for another employment.

Answer: A. Identify the source of the conflict and understand the points of friction This involves a problem solving approach, which addresses the root cause of the problem.

She wants to ensure that every task is carried out as planned. Which of the following tasks is NOT included in the controlling process? A. Instructing the members of the standards committee to prepare policies B. Reviewing the existing policies of the hospital C. Evaluating the credentials of all nursing staff D. Checking if activities conform to schedule

Answer: A. Instructing the members of the standards committee to prepare policies Instructing the members involves a directing function.

When she presents the nursing procedures to be followed, she refers to what type of standards? A. Process B. Outcome C. Structure D. Criteria

Answer: A. Process Process standards include care plans, nursing procedure to be done to address the needs of the patients.

He plans for his sampling method. Which sampling method gives equal chance to all units in the population to get picked? A. Random B. Accidental C. Quota D. Judgment

Answer: A. Random Random sampling gives equal chance for all the elements in the population to be picked as part of the sample.

Which criteria refer to the ability of the instrument to detect fine differences among the subjects being studied? A. Sensitivity B. Reliability C. Validity D. Objectivity

Answer: A. Sensitivity Sensitivity is an attribute of the instrument that allow the respondents to distinguish differences of the options where to choose from

Ms. Valencia is responsible to the number of personnel reporting to her. This principle refers to: A. Span of control B. Unity of command C. Carrot and stick principle D. Esprit d' corps

Answer: A. Span of control Span of control refers to the number of workers who report directly to a manager.

She plans of assigning competent people to fill the roles designed in the hierarchy. Which process refers to this? A. Staffing B. Scheduling C. Recruitment D. Induction

Answer: A. Staffing Staffing is a management function involving putting the best people to accomplish tasks and activities to attain the goals of the organization.

Which of the following is evidence that the controlling process is effective? A. The things that were planned are done B. Physicians do not complain. C. Employees are contended D. There is an increase in customer satisfaction rate.

Answer: A. The things that were planned are done Controlling is defined as seeing to it that what is planned is done.

Which of the following terms refer to the degree to which an instrument measures what it is supposed to be measure? A. Validity B. Reliability C. Meaningfulness D. Sensitivity

Answer: A. Validity Validity is ensuring that the instrument contains appropriate questions about the research topic

He plans to use a Likert Scale to determine A. degree of agreement and disagreement B. compliance to expected standards C. level of satisfaction D. degree of acceptance

Answer: A. degree of agreement and disagreement Likert scale is a 5-point summated scale used to determine the degree of agreement or disagreement of the respondents to a statement in a study.

Which of the following articles would Kevin least consider for his review of literature? A. "Story-Telling and Anxiety Reduction Among Pediatric Patients" B. "Turnaround Time in Emergency Rooms" C. "Outcome Standards in Tertiary Health Care Institutions" D. "Environmental Manipulation and Client Outcomes"

Answer: B. "Turnaround Time in Emergency Rooms" The article is for pediatric patients and may not be relevant for adult patients.

Which of the following statements refers to criteria? A. Agreed on level of nursing care B. Characteristics used to measure the level of nursing care C. Step-by-step guidelines D. Statement which guide the group in decision making and problem solving

Answer: B. Characteristics used to measure the level of nursing care Criteria are specific characteristics used to measure the standard of care.

Alexandra is tasked to organize the new wing of the hospital. She was given the authority to do as she deems fit. She is aware that the director of nursing has substantial trust and confidence in her capabilities, communicates through downward and upward channels and usually uses the ideas and opinions of her staff. Which of the following is her style of management? A. Benevolent -authoritative B. Consultative C. Exploitive-authoritative D. Participative

Answer: B. Consultative A consultative manager is almost like a participative manager. The participative manager has complete trust and confidence in the subordinate, always uses the opinions and ideas of subordinates and communicates in all directions.

She tries to design an organizational structure that allows communication to flow in all directions and involve workers in decision making. Which form of organizational structure is this? A. Centralized B. Decentralized C. Matrix D. Informal

Answer: B. Decentralized Decentralized structures allow the staff to make decisions on matters pertaining to their practice and communicate in downward, upward, lateral and diagonal flow.

Raphael is interested to learn more about transcultural nursing because he is assigned at the family suites where most patients come from different cultures and countries. Which of the following designs is appropriate for this study? A. Grounded theory B. Ethnography C. Case study D. Phenomenology

Answer: B. Ethnography Ethnography is focused on patterns of behavior of selected people within a culture

St. Raphael Medical Center just opened its new Performance Improvement Department. Ms. Valencia is appointed as the Quality Control Officer. She commits herself to her new role and plans her strategies to realize the goals and objectives of the department. Which of the following is a primary task that they should perform to have an effective control system? A. Make an interpretation about strengths and weaknesses B. Identify the values of the department C. Identify structure, process, outcome standards & criteria D. Measure actual performances

Answer: B. Identify the values of the department Identify the values of the department will set the guiding principles within which the department will operate its activities

Which phase of the employment process includes getting on the payroll and completing documentary requirements? A. Orientation B. Induction C. Selection D. Recruitment

Answer: B. Induction This step in the recruitment process gives time for the staff to submit all the documentary requirements for employment.

She notes that there is an increasing unrest of the staff due to fatigue brought about by shortage of staff. Which action is a priority? A. Evaluate the overall result of the unrest B. Initiate a group interaction C. Develop a plan and implement it D. Identify external and internal forces.

Answer: B. Initiate a group interaction Initiate a group interaction will be an opportunity to discuss the problem in the open.

As a young manager, she knows that conflict occurs in any organization. Which of the following statements regarding conflict is NOT true? A. Can be destructive if the level is too high B. Is not beneficial; hence it should be prevented at all times C. May result in poor performance D. May create leaders

Answer: B. Is not beneficial; hence it should be prevented at all times Conflicts are beneficial because it surfaces out issues in the open and can be solved right away. Likewise, members of the team become more conscientious with their work when they are aware that other members of the team are watching them.

The nursing theorist who developed transcultural nursing theory is A. Dorothea Orem B. Madeleine Leininger C. Betty Newman D. Sr. Callista Roy

Answer: B. Madeleine Leininger Madeleine Leininger developed the theory on transcultural theory based on her observations on the behavior of selected people within a culture.

Which pattern of nursing care involves the care given by a group of paraprofessional workers led by a professional nurse who take care of patients with the same disease conditions and are located geographically near each other? A. Case method B. Modular nursing C. Nursing case management D. Team nursing

Answer: B. Modular nursing Modular nursing is a variant of team nursing. The difference lies in the fact that the members in modular nursing are paraprofessional workers.

Aubrey thinks about primary nursing as a system to deliver care. Which of the following activities is NOT done by a primary nurse? A. Collaborates with the physician B. Provides care to a group of patients together with a group of nurses C. Provides care for 5-6 patients during their hospital stay. D. Performs comprehensive initial assessment

Answer: B. Provides care to a group of patients together with a group of nurses This function is done in team nursing where the nurse is a member of a team that provides care for a group of patients.

She checks the documentary requirements for the applicants for staff nurse position. Which one is NOT necessary? A. Certificate of previous employment B. Record of related learning experience (RLE) C. Membership to accredited professional organization D. Professional identification card

Answer: B. Record of related learning experience (RLE) Record of RLE is not required for employment purposes but it is required for the nurse's licensure examination.

He checks if his instruments meet the criteria for evaluation. Which of the following criteria refers to the consistency or the ability to yield the same response upon its repeated administration? A. Validity B. Reliability C. Sensitivity D. Objectivity

Answer: B. Reliability Reliability is repeatability of the instrument; it can elicit the same responses even with varied administration of the instrument

Ms. Valencia develops the standards to be followed. Among the following standards, which is considered as a structure standard? A. The patients verbalized satisfaction of the nursing care received B. Rotation of duty will be done every four weeks for all patient care personnel. C. All patients shall have their weights taken recorded D. Patients shall answer the evaluation form before discharge

Answer: B. Rotation of duty will be done every four weeks for all patient care personnel. Structure standards include management system, facilities, equipment, materials needed to deliver care to patients. Rotation of duty is a management system.

He thinks of an appropriate theoretical framework. Whose theory addresses the four modes of adaptation? A. Martha Rogers B. Sr. Callista Roy C. Florence Nightingale D. Jean Watson

Answer: B. Sr. Callista Roy Sr. Callista Roy developed the Adaptation Model which involves the physiologic mode, self-concept mode, role function mode and dependence mode

These vaccines are given 3 doses at one month intervals: A. DPT (Diphtheria, Pertussis, Tetanus), BCG (Bacillus Calmette-Guérin), TT (Tetanus Toxoid) B. DPT (Diphtheria, Pertussis, Tetanus), TT (Tetanus Toxoid), OPV (oral polio vaccine) C. OPV (oral polio vaccine), Hep. B, DPT (Diphtheria, Pertussis, Tetanus) D. Measles, OPV (oral polio vaccine), DPT (Diphtheria, Pertussis, Tetanus)

Answer: C

A 43 yo female 12 weeks pregnant presents to her OB-GYN for HIV testing following recent knowledge of her husband's infidelity. She tests positive and becomes increasingly emotional stating "There is no point in me continuing this pregnancy since my baby will be born HIV positive." Given the client's statement what is the best response by the nurse? A. If that is how you feel we can discuss different options for abortion. B. While it is true your baby will be born HIV positive, this doesn't mean him or her can't live a happy life. C. If we start you on a series of medications called antiretrovirals and you stick to your prescribed treatment there is a very small chance your baby will be born HIV positive. D. There is no reason to worry, your baby can only become HIV positive if he or she is breastfed once born.

Answer: C Taking treatment properly can reduce the risk of your baby being born with HIV to less than 1%. If you found out that you are HIV-positive before you got pregnant, you may be taking treatment already. If you are not, talk to a healthcare professional about starting treatment straight away. If you found out that you are HIV-positive during your pregnancy, it is recommended that you start treatment immediately and continue taking it every day for life. Your baby will also be given treatment for four to six weeks after they are born to help prevent an HIV infection developing.

Kevin is a member of the Nursing Research Council of the hospital. His first assignment is to determine the level of patient satisfaction on the care they received from the hospital. He plans to include all adult patients admitted from April to May, with average length of stay of 3-4 days, first admission, and with no complications. Which of the following is an extraneous variable of the study? A. Date of admission B. Length of stay C. Age of patients D. Absence of complications

Answer: C. Age of patients An extraneous variable is not the primary concern of the researcher but has an effect on the results of the study. Adult patients may be young, middle or late adult.

Katherine tells one of the staff, "I don't have time to discuss the matter with you now. See me in my office later" when the latter asks if they can talk about an issue. Which of the following conflict resolution strategies did she use? A. Smoothing B. Compromise C. Avoidance D. Restriction

Answer: C. Avoidance This strategy shuns discussing the issue head-on and prefers to postpone it to a later time. In effect the problem remains unsolved and both parties are in a lose-lose situation.

He opts to use a self-report method. Which of the following is NOT TRUE about this method? A. Most direct means of gathering information B. Versatile in terms of content coverage C. Most accurate and valid method of data gathering D. Yields information that would be difficult to gather by another method

Answer: C. Most accurate and valid method of data gathering The most serious disadvantage of this method is accuracy and validity of information gathered

Which of the following statements is NOT true about performance appraisal? A. Informing the staff about the specific impressions of their work help improve their performance. B. A verbal appraisal is an acceptable substitute for a written report C. Patients are the best source of information regarding personnel appraisal. D. The outcome of performance appraisal rests primarily with the staff.

Answer: C. Patients are the best source of information regarding personnel appraisal. The patient can be a source of information about the performance of the staff but it is never the best source. Directly observing the staff is the best source of information for personnel appraisal.

Ms. Valencia prepares the process standards. Which of the following is NOT a process standard? A. Initial assessment shall be done to all patients within twenty four hours upon admission. B. Informed consent shall be secured prior to any invasive procedure C. Patients' reports 95% satisfaction rate prior to discharge from the hospital. D. Patient education about their illness and treatment shall be provided for all patients and their families.

Answer: C. Patients' reports 95% satisfaction rate prior to discharge from the hospital. This refers to an outcome standard, which is a result of the care that is rendered to the patient.

In a horizontal chart, the lowest level worker is located at the A. Left most box B. Middle C. Right most box D. Bottom

Answer: C. Rightmost box The leftmost box is occupied by the highest authority while the lowest level worker occupies the rightmost box.

Which of the following variables will he likely EXCLUDE in his study? A. Competence of nurses B. Caring attitude of nurses C. Salary of nurses D. Responsiveness of staff

Answer: C. Salary of nurses Salary of staff nurses is not an indicator of patient satisfaction, hence need not be included as a variable in the study.

There are times when Katherine evaluates her staff as she makes her daily rounds. Which of the following is NOT a benefit of conducting an informal appraisal? A. The staff member is observed in natural setting. B. Incidental confrontation and collaboration is allowed. C. The evaluation is focused on objective data systematically. D. The evaluation may provide valid information for compilation of a formal report.

Answer: C. The evaluation is focused on objective data systematically. Collecting objective data systematically can not be achieved in an informal appraisal. It is focused on what actually happens in the natural work setting.

A nurse in a health care clinic is preparing to conduct a nutritional session with a group of culturally diverse pregnant women. At the first session the nurse will be meeting with each client individually. The nurse prepares a list of items to be included in the session and lists which item as the first priority? A. Review the MyPyramid food guide. B. Weigh each client and ask the client to document the weight on a progress chart. C. Discuss the costs of food items. D. Identify the food preferences and methods of food preparation for each client.

Answer: D In order to determine each client's nutritional status and needs, the first priority of the nurse is to identify each client's food preferences. Cultural background and knowledge about nutrition are important factors influencing food choices and nutritional status. Although options 1,2 and 3 may be a component of the sessions, option 4 is the first priority.

She knows that performance appraisal consists of all the following activities EXCEPT: A. Setting specific standards and activities for individual performance. B. Using agency standards as a guide. C. Determine areas of strength and weaknesses D. Focusing activity on the correction of identified behavior.

Answer: D. Focusing activity on the correction of identified behavior. Performance appraisal deal with both positive and negative performance; is not meant to be a fault-finding activity

The following are basic steps in the controlling process of the department. Which of the following is NOT included? A. Measure actual performance B. Set nursing standards and criteria C. Compare results of performance to standards and objectives D. Identify possible courses of action

Answer: D. Identify possible courses of action This is a step in a quality control process and not a basic step in the control process.

Kathleen knows that one of her staff is experiencing burnout. Which of the following is the best thing for her to do? A. Advise her staff to go on vacation. B. Ignore her observations; it will be resolved even without intervention C. Remind her to show loyalty to the institution. D. Let the staff ventilate her feelings and ask how she can be of help.

Answer: D. Let the staff ventilate her feelings and ask how she can be of help. Reaching out and helping the staff is the most effective strategy in dealing with burn out. Knowing that someone is ready to help makes the staff feel important; hence her self-worth is enhanced.

She decides to have a decentralized staffing system. Which of the following is an advantage of this system of staffing? A. greater control of activities B. Conserves time C. Compatible with computerization D. Promotes better interpersonal relationship

Answer: D. Promotes better interpersonal relationship Decentralized structures allow the staff to solve decisions by themselves, involve them in decision making; hence they are always given opportunities to interact with one another.

She conducts a 6-month performance review session with a staff member. Which of the following actions is appropriate? A. She asks another nurse to attest the session as a witness. B. She informs the staff that she may ask another nurse to read the appraisal before the session is over. C. She tells the staff that the session is manager-centered. D. The session is private between the two members.

Answer: D. The session is private between the two members. The session is private between the manager and the staff and remains to be so when the two parties do not divulge the information to others.

She decides to illustrate the organizational structure. Which of the following elements is NOT included? A. Level of authority B. Lines of communication C. Span of control D. Unity of direction

Answer: D. Unity of direction Unity of direction is a management principle, not an element of an organizational structure.

5. A famous rock star has just been admitted to Unit 12A after an automobile accident. A nurse on Unit 12B who is a fan of the musician uses the electronic health record (EHR) to find out how the patient is doing. Which is true regarding the use of a patient's EHR? a. Only staff caring for the patient should access this record. b. Permission from a supervisor is needed to read this record. c. The patient's record can be discussed with the nurse's co-worker. d. The nurse can call a friend who works at the local newspaper.

Answer: a Patient information should be accessed only by staff caring for that patient. Security codes are required for EHR access, and access of records can be monitored. Professional ethics should dictate the nurse's behavior, and only the records of patients being cared for should be accessed. A patient's record can be discussed only with those who are caring for the patient, and, because the nurse is not on the same unit, the records should not be accessed or discussed by that nurse. Health Insurance Portability and Accountability Act (HIPAA) laws prevent the discussion of private patient information with anyone outside of the team providing care.

6. Which activity by a unit nurse demonstrates information literacy? a. Researching a patient's diagnosis online b. Entering patient data into the electronic health record (EHR) c. Organizing patient data to study trends d. Learning a new electronic health record system

Answer: a The nurse is demonstrating information literacy (the ability to recognize when information is needed and to locate and use that information) when researching a patient's diagnosis online. Entering patient data into the EHR or learning a new EHR system demonstrates beginner nursing informatics competency. Organizing patient data to study trends demonstrates an experienced level of nursing informatics competency.

2. Which description is an example of data? a. A print-out of a patient's history and physical examination b. A patient's blood pressure and pulse rate c. The nurse's knowledge of a disease d. A nurse's interpretation of a change in the patient's condition

Answer: b Data are facts, observations, and measurements such as blood pressure and pulse rate. A printout of the patient's history and physical is organized information that is meaningful. Knowledge is organized and processed information such as a nurse's knowledge of a disease. When nurses interpret a change in the patient's condition, they are using wisdom or the use of knowledge and experience to manage and solve problems.

4. The hospital has implemented a new electronic medication administration record (MAR). What is true about the use of this new tool? a. Verifies medication dosages b. Reduces medication administration errors c. Eliminates the need to count narcotics d. Requires a hard copy of the MAR to be printed

Answer: b The electronic MAR reduces medication errors by requiring the nurse to scan the patient's identification band and the medication. Although the electronic MAR alerts the nurse to potential errors such as the wrong dose, it is the nurse's responsibility to verify all information before administration of a medication. Narcotic counts are still kept in the electronic system. A hard copy of the MAR is not necessary.

7. The nurse is assigned to administer medications to a patient on a unit that has just implemented bar code medication administration (BCMA). Which step is proper for the nurse to follow? a. Open the medication packages at the nurses' station. b. Ask the patient to verify his or her address. c. Scan the nurse's ID, the patient's ID, and the code on the medication package. d. Ask the patient to name two patient identifiers.

Answer: c The BCMA system scans the nurse's ID, the patient's ID, and the medication package to ensure that the proper drug is given to the correct patient. Asking the patient's address or two random identifiers that the patient may not be aware of would be inappropriate. Proper protocol for administration is to open the medication packages at the bedside. Use of a scanning device requires the medication to still be in the package while scanning to ensure that it is the proper medication.

10. Which description is true about the Nursing Minimum Data Set (NMDS)? a. An admission assessment tool b. A discharge summary c. The core nursing data for collection across all sites d. An organization of nursing diagnoses

Answer: c The NMDS is a standardized collection of essential nursing data used by nurses to promote consistent, understandable documentation. Although standard terminology may be used during admission, and discharge, these tools are not a description of NMDS. Nursing diagnoses are organized using NANDA-I.

8. Which statement is correct concerning the implementation of computerized provider order entry (CPOE)? a. The unit secretary transcribes the physician's orders into the computer. b. The nurse must ensure that orders go to the appropriate departments. c. Physician orders go directly to the appropriate department. d. Handwriting legibility is a major problem.

Answer: c Use of CPOE enables orders to go directly to the appropriate department decreasing the potential for errors. There is no transcription of orders and no need for someone to transcribe the orders. Because the orders are typed into the computer, handwriting legibility is not an issue.

3. Which items are supported by point-of-care use of information technology? (Select all that apply.) a. More accurate documentation b. Direct access to diagnostic results c. Confidentiality d. Direct access to records by patients e. Access to medication profiles

Answers: a, b, c, e Point-of-care use of information technology provides more accurate documentation because the nurse documents patient information in real time. Diagnostic results are immediately available to the nurse. Confidentiality is protected when the nurse documents at the bedside using a secure log-in and password. Medication profiles are available to the nurse at bedside. Patients do not have direct access to their medical records because these records are secured by log-ins and passwords accessible to only health care providers caring for each patient.

9. Which behaviors are expected of the nurse at the experienced informatics competency level? (Select all that apply.) a. Collect accurate assessment data. b. Conduct informatics research. c. Group assessment data. d. Document data appropriately on the electronic health record (EHR). e. Integrate information science, computer science, and nursing science.

Answers: a, c, d The nurse at the intermediate level of informatics competency can see data relationships and is able to collect and group data. The nurse is skilled in the use of computer technology and can document in the EHR. Conducting informatics research and integration of the sciences is the advanced level of informatics competency.

1. Which descriptions are advantages of health care information technology (IT)? (Select all that apply.) a. Increases health care delivery costs b. Improves communication among providers c. Improves administration functions d. Increases time necessary to document care e. Decreases the safety of providing care

Answers: b, c Health care IT improves communication among providers by providing immediate access that is legible and standardized to patient data by all providers. IT improves administrative functions by addressing the issues of quality, cost-effectiveness, and outcomes of care. Although there are initial costs to purchase hardware and software, these systems are cost-effective in the long run. Systems that support data collection at the point of care can directly enhance patient care by decreasing the time spent on documentation, reducing the potential for errors, and supporting improved assessment and data communication

An organization that sets standards of practice for nursing is the: A. American Medical Association (AMA) B. American Nurses Association (ANA) C. utilization review committee D. American Academy of Pediatrics

B. American Nurses Association (ANA) Rationale: The ANA develops standards of practice. (3)

Choose the best description for a variance in a clinical pathway. A. The patient did not cooperate with the recommended therapy. B. An achieved patient outcome differs from the expected outcome. C. Patient care is individualized, appropriate to a specific person. D. Reimbursement will be curtailed because of a complication.

B. An achieved patient outcome differs from the expected outcome. Rationale: Clinical pathways provide "targets" for expected progress along a timeline and are often collaborative, using skills of multiple professionals. When the patient "falls off" of the expected course, a variance occurs. Because it is about outcomes, variance the clinical pathway does not refer to issues of noncompliance, reimbursement issues, or the tailoring of a treatment plan to best fit that individual. (12)

Which is a diagnosis that the nurse is legally responsible to report? A. Bronchitis B. Child abuse C. Acute otitis media D. Strep throat

B. Child abuse Rationale: All nurses have legal and ethical responsibility to report child or elder abuse.

The nurse was just hired to work in a hospice day care facility. She reflects on her ethical framework and decides that she believes in establishing relationships with an emphasis on caring. Which frameworks can the nurse use? Select all that apply. A.Deontology B.Feminist ethics C.Utilitarianism D.Ethics of care

B. Feminist ethics D. Ethics of care

1. What is the ultimate goal of increasing the parasympathetic system influence on the body through relaxation or meditation? A. Stimulate improved gastrointestinal function B. Reduce the effects of stress on the body C. Increase cardiac output and blood pressure D. Facilitate respiratory function and cough

B. Reduce the effects of stress on the body

Trina, an RN on the nursing unit, states to the student nurse, "I see you are working with Beth today. Make sure you watch your patient carefully because I think Beth is one of our worst nurses." The student knows that this statement could be considered A.Libel B.Slander C.Fraud D.Invasion of privacy

B. Slander

The new graduate RN needs to have a preceptor assigned to him as he begins working in the pediatric clinic. The BEST person for this assignment would be A.The charge RN who is a natural leader and knows all of the policies. B.The staff RN who has 3 years of experience and enjoys orienting new employees. C.The LPN with 15 years of experience who knows all of the doctors' preferences. D.The staff RN with 10 years of experience who refuses to work with students.

B. The staff RN who has 3 years of experience and enjoys orienting new employees.

The most important nursing action to prevent infection in any patient is to: A. use disposable equipment. B. consistently perform hand hygiene. C. limit visitors to family members. D. wear hospital-laundered clothes.

B. consistently perform hand hygiene. Rationale: Because the nurse must handle equipment, dressings, and patient care activities, both with and without gloves, consistent hand hygiene reduces the number of resident bacteria that can be transmitted o others who may not be resistant to infection. Naturally, disposable equipment is not always practical. Finally, if good hand hygiene is practiced, visitor limitation and hospital-laundering and become secondary concerns; none of these are on a par with hand hygiene. (2)

The nurse is completing an admission assessment. Which interview question best reflects a belief related to holistic healthcare? A."Can you tell me the names of the medications you are taking?" B."Can you tell me how you normally handle stress?" C."Can you tell me how long you have had your symptoms?" D."Can you tell me your age and where you live?" §

B."Can you tell me how you normally handle stress?"

1. After a stroke, a client is having difficulty swallowing. The nurse may make a referral to what member of the health care team? A. Physical therapist B. Respiratory therapist C. Speech therapist D. Social worker

C. Speech therapist

A group of women are discussing childbirth experiences. Which statement would most likely indicate that the woman gave birth in the 1950s? A. "My husband stayed with me throughout labor and birth." B. "The suite allowed me to deliver and recover in the same room" C. "I was discharged from the hospital 1 week following deliver." D. "The birthing center rooms were decorated in a homelike fashion."

C. "I was discharged from the hospital 1 week following deliver." Rationale: During the 1950s, the hospital stay for labor and delivery was 1 week. The other situations described would not have occurred until after 1960 with the natural childbirth movement. (6)

The nurse admitting a 3-year-old patient with the medical diagnosis of pneumonia identifies congestion and inspiratory wheezes in both lungs. This information is considered part of which step of the nursing process? A. Diagnosis B. Evaluation C. Assessment D. Implementation

C. Assessment Rationale: Assessment involves collection of patient data. Diagnosis, implementation, and evaluation are other steps in the nursing process that involve analysis of data, carrying out interventions and evaluating outcome progress. (11)

Which government department is responsible for overseeing the administration of the WIC program? A. Social Security Administration B. Department of Health and Human Services C. Department of Public Health D. Children's Bureau

C. Department of Public Health Rationale: This department is responsible for improving the morbidity and mortality of children. The others do not have this responsibility.

The client has right-sided hemiplegia as a result of a stroke (brain attack) and wants a cup of hot coffee. Even though the client is insistent, the nurse does not permit her to drink the coffee unsupervised. This nurse is using the ethical principle of A.Autonomy B.Fidelity C.Non-maleficence D.Justice

C. Non-maleficence

The nursing process is best described as method to: A. identify patients who have an increased risk for medical complications. B. reduce the incidence of complications for expectant mothers and infants. C. identify and solve patient problems with individualized nursing care. D. promote breastfeeding in groups that do not usually nurse infants.

C. identify and solve patient problems with individualized nursing care. Rationale: The nursing process helps nurses individualize care for all patients, both well and ill. While this may include tasks identifier in the other choices, including helping to identify risks, working collaboratively to reduce complications, and promoting breastfeeding through public health outreach, none of these best describes the function of the nursing process as a whole. (11)

To best improve the care of a pregnant woman from a different cultural group, the nurse should: A. identify the woman's expectations about pregnancy and birth. B. observe the woman and her family as they interact. C. learn about members of different local cultural groups. D. encourage the woman to adopt local childbearing practices.

C. learn about members of different local cultural groups. Rationale: Most nurses can identify major cultural groups that reside in the area where they work and gain an understanding of their major health care beliefs and values. Patients and their families appreciate the extra attention to incorporate cultural needs into other nursing care. While it's always constructive to identify the woman's expectations about pregnancy and birth, this does not particularly aid in understanding the cultural beliefs as a whole. Observing one family's interactions is not only intrusive, but also unnecessary with a good understanding of cultural distinctions and preferences. Finally it can be very insensitive to request that a woman adopt childbearing practices that are foreign to her and may, in some cases contradict highly valued beliefs and traditions. (6, 7, Nursing Care Plan 1-1)

The new charge RN on a hospital unit is leading a committee that must choose new paint colors for the nurses' station. She elicits the opinions of all group members and then organizes a vote. The charge nurse's leadership style can be said to be A.Laizzez-faire B.Autocratic C.Democratic D.Scientific

C.Democratic

Acupuncture

Chinese medical practice of inserting needles into certain areas of the body

The relative health and well-being of the nurse in relation to helping patients constitutes a vital force in the healing process. A method that would be helpful to nurses in fostering their own health is: Increase stress Clarify values and beliefs Work and act in isolation Setting unrealistic goals

Clarify values and beliefs

What are the effects of massage as a manual healing method? Select all that apply. Communication and caring Mental and physical relaxation Increased muscle strength Speeds the removal of waste products Lowers blood pressure and heart rate

Communication and caring Mental and physical relaxation Speeds the removal of waste products Lowers blood pressure and heart rate

1. A client inquires about the use of herbal therapy. Which statement by the nurse is most accurate? A. "Be sure to pay attention to the packaging's therapeutic and prevention information." B. "All herbs are equal in purity, so purchase the cheapest brand." C. "It is best if you select a licensed herbalist as a practitioner." D. "Herbs can have side effects and can interact with prescription medications."

D. "Herbs can have side effects and can interact with prescription medications."

Choose the best description of certified nurse-midwife (CNM) qualifications. A. Gives limited care to women after normal childbirth. B. Assists in the prenatal care of high-risk women. C. Primarily provides care to low-income women. D. Attends uncomplicated births of low-risk women.

D. Attends uncomplicated births of low-risk women. Rationale: The CNM is an advanced-practice nurse with special training in providing care to the childbearing woman; attending births that are expected to be uncomplicated certainly falls within that realm. Beyond that, the CNM works under protocols that specify under what conditions that medical care must be sought for the woman. At the same time, CNMs are not limited to post childbirth care, nor do they necessarily provide care primarily to low-income women. Finally, a CNM would work collaboratively with a specialist or multi-disciplinary team in prenatal care of a woman at high risk for complications. (6)

Which nursing action is the priority when a patient arrives at the clinic? A. Prepare a plan of care B. Select the appropriate NANDA diagnoses C. Administer medications as ordered D. Determine and document history and vital signs

D. Determine and document history and vital signs Rationale: Assessment is the first step of the nursing process and included the collection of subjective and objective patient data. (11)

On admission to Hospital A, the patient indicates that he would like information about making a living will. The nurse informs the patient that the hospital does not have a process in place to facilitate this. Hospital A is not in compliance with the A.Health Insurance Portability and Accountability Act B.Emergency Medical Treatment and Active Labor Act C.Americans with Disabilities Act D.Patient Self-Determination Act

D. Patient Self-Determination Act

Which source would the nurse use to determine whether a specific nursing activity is within the scope of practice of an LPN/LVN? A. Doctor's prescription record B. Nursing procedure manual C. Head nurse or nurse manager D. The Nurse Practice Act

D. The Nurse Practice Act Rationale: Specific activities are listed by the state nurse practice act and nurses must practice within the limitations of the nurse practice act for their state. The other answer options are not sources that determine scope of practice. (3)

The nurse is providing "evidence based care" when she: A. adheres to hospital procedure book guidelines. B. carries out protocols learned in nursing school. C. carries out the practice requested by the charge nurse or health care provider. D. knows that the practice has been published in a professional journal or text.

D. knows that the practice has been published in a professional journal or text. Rationale: Evidence-based practice starts when the nurse uses the best evidence obtained from current, valid, published research. When the nurse combines that information with his or her critical thinking process, experiences, and patients' needs, it is then possible to plan safe, effective nursing care for the patient.(14)

The number of deaths of infants younger than age 28 days per 1000 live births is termed the: A. infant death rate B. neonatal birth rate C. neonatal morbidity rate D. neonatal mortality rate

D. neonatal mortality rate Rationale: Neonatal mortality rate is defined as the number of deaths of infants under age 1 year per 1000 live births. Birth rate and neonatal birth rate refer to the number of births per 1000 population in a year. Neonatal morbidity rate refers to illness. (12, Box 1-6)

A nursing unit is now doing a walking change-of-shift report. The unit manager puts out a survey asking for employee feedback about the new process. This would be termed: A.Overcoming resistance B.Unfreezing C.Implementing change D.Integrating change

D.Integrating change

Jason just vomited blood but is hesitant to go to the emergency department because he does not have insurance. He tells this to the admitting nurse, who assures him he won't be turned away from medical care.

Emergency Medical Treatment and Active Labor Act (EMTALA)

Which nursing action is most likely to create a healing environment? Use technology to prevent hospital-acquired infections. Empower clients to make healthy decisions for themselves. Assist clients to obtain a safe and comfortable place to live. Ensure that physicians' orders are carried out.

Empower clients to make healthy decisions for themselves.

Common malpractice claims

Failure to assess and diagnose Failure to plan Failure to implement a plan of care Failure to evaluate

Henry, a nurse, is driving home from work when there is a major motor vehicle accident in front of him. He runs to the side of the driver, finding him bleeding from the nose and mouth. Henry calls 911 and begins treating the driver after receiving consent.

Good Samaritan Law

The nurse performs care on an unconscious person at the grocery store. What law protects the nurse in this situation? Medical Treatment and Active Labor Act American Nurses Association Code of Ethics Good Samaritan laws Nurse practice acts

Good Samaritan laws

Era II, "mind--body" medicine arose in the mid-1950s and still has some influence in the world today. These therapies focus on helping individuals to use their minds to _____ their bodies. Heal Cure Trans-personalize Ignore

Heal

Michelle is in grave condition after an automobile accident. The nurse knows he cannot give a status update to her sister calling from out of state. What is this an example of? Choices for Questions 1-10 Health Insurance Portability and Accountability Act (HIPAA) Emergency Medical Treatment and Active Labor Act (EMTALA) Patient Self-Determination Act (PSDA) Americans With Disabilities Act (ADA) Good Samaritan Law American Nurses Association Code of Ethics Patient Care Partnership Standard of Practice Nurse Practice Acts Scope of Practice

Health Insurance Portability and Accountability Act (HIPAA)

Through the use of the nursing process, holistic health care rendered by nurses includes all but one of the stated activities: Health promotion Health maintenance Illness accentuation Restorative rehabilitative care.

Illness accentuation

From the perspective of traditional Chinese medicine, which is the best definition of disease? Imbalance or disruption in food digestion. Imbalance or interruption in the flow of qi. Imbalance or disruption in key social relationships. Imbalance or disruption in thoughts or emotions.

Imbalance or interruption in the flow of qi.

Meditation

Influences the parasympathetic system on body which helps reduce the stress on the body.

A client asks the nurse to state one of the primary principles associated with naturopathy. Which of the following is the best response? A higher being guides the learning needed to treat disease. It focuses on environmental causes when treating illnesses. It focuses on early detection and treatment of disease. It is a way of life to maintain health and prevent disease.

It is a way of life to maintain health and prevent disease.

Which oil may be placed directly on the skin? Rose and orange Green apple and jasmine Clary sage and rosemary Lavender and tea tree

Lavender and tea tree

What is leadership?

Leadership is the process of influencing people by providing purpose, direction, and motivation to accomplish the mission and improve the organization.

You have been asked to share information about music therapy with a group of high school students who are studying the effects of alternative treatment methods in health care. Which statement below most accurately reflects strength of this therapy? Music can serve as a vehicle for altering ordinary levels of consciousness to achieve the mind's fullest potential. Music therapy can only be used in a limited practice setting. Any type of music will help the body restore regulatory functions in times of stress or illness. Only the right side of the brain is active when utilizing this therapy.

Music can serve as a vehicle for altering ordinary levels of consciousness to achieve the mind's fullest potential.

Annie is planning to move to another state. She looks at the Board of Nursing website to explore that state's regulations for registered nurses.

Nurse Practice Acts

Johnston would like to better understand his hospital bill. He calls the hospital and the billing department suggests he meet with a representative and have it explained.

Patient Care Partnership

A client is asking about developing a living will. What act protects this right? Americans With Disabilities Act Emergency Medical Treatment and Active Labor Act Patient Self-Determination Act Health Insurance Portability and Accountability Act Submit

Patient Self-Determination Act

Josephine wants to make her own decisions about her end-of-life care. She talks to her significant other about creating a living will and durable power of attorney.

Patient Self-Determination Act (PSDA)

Nurses assist in performing relaxation, biofeedback, and imagery techniques. These techniques share the following processes: Physical resting and rhythmic breathing Therapeutic touch and meditation Physical activity and therapeutic touch Physical resting and reflexology

Physical resting and rhythmic breathing

The nurse is explaining the Health Insurance Portability and Accountability Act to a group of new employees. What should the nurse include when explaining its purpose? Select all that apply. Protects health insurance benefits Provides transferability of insurance to others Protects family members Protects those with preexisting conditions Provides personal health information privacy

Protects health insurance benefits Protects those with preexisting conditions Provides personal health information privacy

Mandatory Reporting Law

Requires certain professionals, such as doctors, to report suspected cases of child abuse

Which guidelines are established for nursing practice? Select all that apply. Manual of Labor Standards of Practice Patient Care Partnership Scope of Responsibility Nurse practice acts

Standards of Practice Patient Care Partnership Nurse practice acts

Herbal products are very popular worldwide and in the United States for prevention and treatment of illness. It is believed that the health practitioner should: Study and include them in questions when taking a patient's history Study them just for the knowledge Not bother studying them Use them for himself

Study and include them in questions when taking a patient's history

You have been invited to talk to the Women's Guild in your community about the cautions of aromatherapy. Which of the following would be correctly presented to the audience? Aromatic oils are produced by a standard quality formula. The oils can be stored in any type of container. The skin should always be tested for allergies by applying a small amount of oil to the area before treatment. Oils should not be used in pregnancy.

The skin should always be tested for allergies by applying a small amount of oil to the area before treatment.

What is a rationale for assessment of clients' use of herbs? There are potential adverse interactions between some herbs and some medications. Clients should not take anything that is not prescribed by the physician. These data will contribute to the body of knowledge on the use of herbs. It is important to establish a pattern that clients tell nurses everything.

There are potential adverse interactions between some herbs and some medications.

Nurses who value client advocacy follow what guideline?

They give priority to the good of the individual client rather than to the good of society in general.

Massage, acupressure, reflexology, and Reiki all share a common characteristic in how their therapeutic intervention is managed. This characteristic involves: Touch therapy Mind--body therapy Music therapy Meditation therapy

Touch therapy

Yoga

a system of exercises practiced as part of the Hindu discipline to promote control of the body and mind

Which are roles of the community health nurse? Select all that apply. a. Case manager b. Advocate d. Counselor e. Facilitator f. Teacher

a. Case manager b. Advocate d. Counselor f. teacher

The nurse is working at a low-income prenatal clinic providing free services, including dietary counseling, exercise, and parenting classes.Which type of nursing is this?Make the Connection. Drag and drop proper term that is best exemplified by each scenario. a. Community Health Nursing b. Public Health Nursing c. Community-Oriented Nursing d. School Nursing e. Occupational Health f. Parish Nursing g. Nursing in Correctional Facilities h. Public Health Clinics i. Disaster Services Nursing j. International Nursing

a. Community Health Nursing Community health nursing focuses on how the health of individual, families, and groups affects the community as a whole. By encouraging a healthy lifestyle for a pregnant mother, both the mom and baby will benefit.

What does a home health nurse do on the first home visit? a. Determine specific skilled care needed and make any referrals. b. Teach clients and caregivers skills, such as measuring dosages, giving injections, and managing intravenous therapy. c. Involve family members in the care of an older adult as needed, including giving medication. d. Educate clients and family members on proper hand hygiene and infection control for the home.

a. Determine specific skilled care needed and make any referrals. First visit should involve an assessment of client needs, equipment needed, interventions, and any referrals needed such as physical therapy, social services, and nutritional support

What is one of the primary goals in home health-care nursing? a. Improving the client's self-care ability and independence b. Providing comfort and assistance with end-of-life care c. Improving health to eliminate the future need for home-based health care d. Monitoring self-administering of medications and self-care for optimal healing

a. Improving the client's self-care ability and independence

When educating the client on care of enteral therapy, what important information should be conveyed? a. Keep opened feeding solutions in the refrigerator until the expiration date. b. Caregivers should purchase small amounts at a time c. Use disposable items and preparation equipment. d. Avoid contaminating the feeding solution by using gloves.

a. Keep opened feeding solutions in the refrigerator until the expiration date. clients may not understand that enteral feedings are food product

Upon approaching a client's home for his or her dressing change, the nurse hears a very hostile argument and glass shattering. What should the nurse do to ensure his or her safety? a. Leave immediately, drive to a safe location, and phone the client to reschedule. b. Knock on the door, announce himself or herself, and wait to see if the argument stops before leaving. c. Return to the car and use the cell phone to call the client and announce the nurse is there. d. Contact the police and then the agency to report possible violence against the client.

a. Leave immediately, drive to a safe location, and phone the client to reschedule. if the nurse feels uncomfortable due to a threat of violence, it is wise to leave and retreat to a safe public place. Once the nurse is safe, he or she can contact the client to reschedule and alert the agency that a security escort may be needed in the future

What leading factors does Healthy People 2020 identify for measuring the health of communities in the United States? Select all that apply. a. Physical activity b. Mental health c. BMI d. Access to health care e. Vulnerability f. Quality of health care

a. Physical activity b. Mental health c. BMI d. Access to health care

The nurse arrives to a client's home to assess a surgical incision and perform a dressing change. The client is nonambulatory with multiple care needs. The nurse recognizes that the spouse, who is the primary caregiver, appears to be physically exhausted and disheveled with a flat affect. What is a possible nursing intervention for the client's spouse? a. Provide information on support groups in the area. b. Refer the spouse to a psychologist. c. Refer the spouse to a psychiatrist. d. Locate a respite care agency.

a. Provide information on support groups in the area. best option for assisting the spouse to adapt to the caregiver role

A family of six has a child with special needs who requires quite a bit of time, attention, and care. They are caring for the child at home but do need relief from time to time. What type of care would they need? a. Respite care b. Direct care agency c. Indirect service agency d. Volunteer organization

a. Respite care

A new grad is exploring the role of the community health nurse. What services might be provided by this person? Select all that apply. (IDK) a. Running a health clinic for uninsured children b. Monitoring polio outbreaks in a geographic location c. Conducting diabetic education at the senior community center d. Trending of community illness e. Performing tuberculosis testing for high-risk homeless individuals

a. Running a health clinic for uninsured children b. Monitoring polio outbreaks in a geographic location c. Conducting diabetic education at the senior community center e. Performing tuberculosis testing for high-risk homeless individuals

What is the main focus of a public health nurse? a. The community as a whole b. Governmental policies and public health education c. Families and individuals of a common demographic d. Groups of individuals and their effect on the community

a. The community as a whole

A nurse is visiting a new home health client who was discharged after partial amputation with a Foley catheter and a history of diabetes. What should the nurse assess for increased risk of infection for this client? Select all that apply. a. Uncontrolled home environment b. Increased use of indwelling devices c. Complexity of illness and care outside of the hospital d. Ineffective reporting and monitoring e. Lack of communication between caregivers

a. Uncontrolled home environment major contribution of infection due to lack of appropriate disinfection b. Increased use of indwelling devices catheters, drainage tubing, and heart valves are common indwelling devices that are common needs of the home health-care clients c. Complexity of illness and care outside of the hospital increases the risk of developing infection d. Ineffective reporting and monitoring can hinder identification of clients at high risk for infections

A graduate nurse is considering taking employment as a community health nurse and is concerned that there may not be adequate resources available to the client population. Which observatory assessment would most help to analyze the community? a. Windshield survey b. Databases/public records c. Client perceptions d. Omaha System

a. Windshield survey gives the researcher an opportunity to directly observe the community in its natural state, including resources, building, public protection, crime and so on

Institutional policies

are guidelines developed by healthcare institutions to direct professional practice.

Personal values

are ideas or beliefs a person considers highly important and are learned through interactions with social systems as described previously.

Professional ethics

are values held by a group deemed as having generalizable application and standards of conduct to be upheld in all situations.

What are common expectations of the community-based nurse? Select all that apply. a. Previous hospital experience b. High level of professionalism c. Nonjudgmental d. Sensitive to differences in people e. Live in the community they serve

b. High level of professionalism c.Nonjudgmental d. Sensitive to differences in people

The nurse is tracking the trend of increased measles outbreaks in state public universities and providing immunization boosters to the campus health clinics.Which type of nursing is this? a. Community Health Nursing b. Public Health Nursing c. Community-Oriented Nursing d. School Nursing e. Occupational health f. Parish Nursing g. Nursing in Correctional Facilities h. Public Health Clinics i. Disaster Services Nursing j. International Nursing

b. public health nursing Public health nursing tracks and monitors communicable outbreaks in the community, providing quick action to prevent the spread of illness.

A new nursing graduate has an interest in a position that will provide health promotion, illness prevention, early detection, and treatment within her rural community.Which type of nursing is this? a. Community Health Nursing b. Public Health Nursing c. Community-Oriented Nursing d. School Nursing e. Occupational Health f. Parish Nursing g. Nursing in Correctional Facilities h. Public Health Clinics i. Disaster Services Nursing j. International Nursing

c. Community-Oriented Nursing Community-oriented nursing encompasses a comprehensive look at the individual, family, group, and community to determine the best plan for improving overall health.

standards of care

can also refer to informal or formal guidelines that are generally accepted in the medical community for treatment of a disease or condition. It may be developed by a specialist society or organization and the title of standard of care awarded at their own discretion. It can be a clinical practice guideline, a formal diagnostic and treatment process a doctor will follow for a patient with a certain set of symptoms or a specific illness

The nurse provides care to children with acute and chronic health conditions while they are attending school each day.Which type of nursing is this? a. Community Health Nursing b. Public Health Nursing c. Community-Oriented Nursing d. School Nursing e. Occupational Health f. Parish Nursing g. Nursing in Correctional Facilities h. Public Health Clinics i. Disaster Services Nursing j. International Nursing

d. School Nursing School nursing takes place in the school system. Children may have chronic health conditions or acute situations the nurse must address. School nurses provide routine vision and hearing screenings and track and monitor growth

14. Which of the following is the biggest advantage of mobile technology in the health care setting? a. improvement in research awareness b. advancement of computer skills c. increased job satisfaction d. prevention of medication errors

d. prevention of medication errors

Code Of Ethics

delineates the conduct and responsibilities expected of all nurses in their nursing practices

A nursing student is studying the principle of autonomy. Which example most accurately depicts this principle

describing surgery to a client before the consent is signed

The nurse works for the automobile industry, providing its workers everything from health screenings to examinations for disability claims.Which type of nursing is this? a. Community Health Nursing b. Public Health Nursing c. Community-Oriented Nursing d. School Nursing e. Occupational Health f. Parish Nursing g. Nursing in Correctional Facilities h. Public Health Clinics i. Disaster Services Nursing j. International Nursing

e. Occupational Health The occupational health nurse provides education and health services for employees and their families to reduce absentee hours and increase productivity. He also provides documentation for disability claims or occupational hazardous events.

Human relations management

focuses on people, particularly the psychological and social aspects of work

This nurse delivers exams, medications, and treatments in the criminal justice system.Which type of nursing is this? a. Community Health Nursing b. Public Health Nursing c. Community-Oriented Nursing d. School Nursing e. Occupational Health f. Parish Nursing g. Nursing in Correctional Facilities h. Public Health Clinics i. Disaster Services Nursing j. International Nursing

g. Nursing in Correctional The correctional nurse works with the community within the criminal justice system. She collaborates with the medical team to provide routine exams and both acute and chronic healthcare on a scheduled or as-needed basis.

This nurse works with the government to provide millions of immunizations at no charge to those within the community.Which type of nursing is this? a. Community Health Nursing b. Public Health Nursing c. Community-Oriented Nursing d. School Nursing e. Occupational Health f. Parish Nursing g. Nursing in Correctional Facilities h. Public Health Clinics i. Disaster Services Nursing j. International Nursing

h. Public Health Clinics Public health clinics often exist within local and state departments of health. They offer discounted illness prevention and wellness services that vary based on the needs of the community.

This nurse is on call to attend to the needs of those in a community experiencing a tragedy.Which type of nursing is this? a. Community Health Nursing b. Public Health Nursing c. Community-Oriented Nursing d. School Nursing e. Occupational Health f. Parish Nursing g. Nursing in Correctional Facilities h. Public Health Clinics i. Disaster Services Nursing j. International Nursing

i. Disaster Services Nursing When a disaster occurs, nurses are called to action. A disaster service nurse must be prepared for the unpredictable, urgent, and uncertain when a disaster such as a flood, earthquake, or terrorist attack occurs.

Guided Imagery

intentional visualization of images that are calming, relaxing, or beneficial in other ways

Personal morality

is the set of beliefs about the standards of right and wrong that help a person determine the correct or permissible action in a given situation.

This nurse works in another country providing relief services after a natural or human-made disaster.Which type of nursing is this? a. Community Health Nursing b. Public Health Nursing c. Community-Oriented Nursing d. School Nursing e. Occupational Health f. Parish Nursing g. Nursing in Correctional Facilities h. Public Health Clinics i. Disaster Services Nursing j. International Nursing

j. International Nursing International or global nursing includes the care of those outside the United States. In addition to disaster response services, this type of nurse cares for those who are malnourished or dehydrated or have insect-related illness, parasitic infections, or HIV.

Professional regulations and laws that govern nursing practice are primarily in place for which reason

to protect the safety of the public

4. What is telehealth nursing?

transmission by a nurse of electronic data, images, or audio from a patient's bedside or home to other health providers for the purpose of providing care and improving outcomes.

Holistic Care

•Uniqueness of each individual •Interaction among mind, body, and spirit •Multiple factors as causes of illness and disease •Patient's participation in treatment and healing

When conducting a weekly home visit with an older client, the nurse realizes the client has not been adhering to his or her recommended medication schedule. What should the nurse do to correct this? a. Investigate why. b. Reprimand the client for being noncompliant and explain how important it is to not miss dosages of medications. c. Explain the effects of missing dosages of certain medications. d. Evaluate the medication schedule.

a. Investigate why. important to understand why the client has not been adhering to the scheduled medication. Issues such as vision, literacy, and side effects could be easily rectified if investigated

The population served by this nurse is the congregation of the church.Which type of nursing is this? a. Community Health Nursing b. Public Health Nursing c. Community-Oriented Nursing d. School Nursing e. Occupational Health f. Parish Nursing g. Nursing in Correctional Facilities h. Public Health Clinics i. Disaster Services Nursing j. International Nursing

f. Parish Nursing The parish nurse serves the church community. His role may include education, counseling, or helping a member navigate the healthcare system


Kaugnay na mga set ng pag-aaral

Topic 4 Test Preparations ( 8th grade US history ) ( texas )

View Set

Foundations of Western Culture14-27

View Set

Organizational Behavior Assignment 5 Chapters 10+11

View Set

Chapter 16 Nursing Management During the Postpartum Period

View Set

PHEO*610*600*TOXICOLOGY IN PUBLIC HEALTH*EXAM I (in class)

View Set